Sei sulla pagina 1di 96

CIVIL LAW REVIEW II

PRACTICE QUESTIONS
AND
SUGGESTED ANSWERS
IN
CIVIL LAW REVIEW II

Obligations
Contracts
Sales
Lease
Credit Transactions
Agency
Trust
Partnership
Torts and Damages

Practice Questions and Suggested Answers 1


CIVIL LAW REVIEW II

INSTRUCTIONS

1. This is merely a compilation of practice questions and suggested answers. You are not supposed
to memorize these questions in lieu of reading cases and commentaries as preparation for your
exams.
2. As stated, these are merely SUGGESTED answers. You are free to disagree and have your own
answer or opinion.
3. This compilation is a mere supplementary material for your Civil Review II class. Do not rely on
this alone. Memorizing this by heart will not guaranty a passing grade for the class.

Practice Questions and Suggested Answers 2


CIVIL LAW REVIEW II

Table of Contents

Obligations: General Principles ..................................................................................................................... 4


Obligations .................................................................................................................................................. 13
Kinds of Obligations .................................................................................................................................... 17
Joint and Solidary Obligations (Computation) ............................................................................................ 22
Modes of Extinguishment ........................................................................................................................... 25
Contracts ..................................................................................................................................................... 30
Status of Contracts ...................................................................................................................................... 36
Sales ............................................................................................................................................................ 37
Status of Contracts ...................................................................................................................................... 58
Sales II ......................................................................................................................................................... 59
Maceda Law ................................................................................................................................................ 62
Status of Contracts of Sale .......................................................................................................................... 64
Remedies..................................................................................................................................................... 66
Extinguishment of Contract of Sale ............................................................................................................ 68
Lease ........................................................................................................................................................... 70
Credit Transactions: General Principles ...................................................................................................... 73
Credit Transactions ..................................................................................................................................... 79
Agency in General ....................................................................................................................................... 91
Agency ......................................................................................................................................................... 94
Partnership in General .............................................................................................................................. 100
Partnership................................................................................................................................................ 104
Torts and Damages in General .................................................................................................................. 106
Torts and Damages ................................................................................................................................... 109

Practice Questions and Suggested Answers 3


CIVIL LAW REVIEW II

OBLIGATIONS: GENERAL PRINCIPLES

TRUE OR FALSE

1. If the debtor obliged to do something fails to do it, an action for specific performance will
prosper.
a. FALSE. It will violate involuntary servitude. The remedy is the obligation may be done
by another at the expense of the debtor.

2. In conditional obligations, the obligation arises upon the happening of the condition.
a. FALSE. There are different kinds of conditional obligations. The condition may be
resolutory.

3. In case of non-compliance of obligations with a penal clause, the penalty shall substitute the
indemnity for damages and the payment of interest.
a. FALSE. The statement is not absolute; an exception is if there was an agreement.

4. The insolvency of one debtor will increase the liability of his co-debtors.
a. FALSE. This does not always apply. The liability will not increase if the obligation is
joint.

5. That obliged to deliver or do something (???) in delay only from the time the obligee judicially or
extra-judicially demands from them the fulfillment of their obligation.
a. FALSE. The statement is not absolute; an exception is when an obligation doesn’t need
demand but there is delay.
i. When stipulated by the parties, by provision of law, when time is the essence
or when the demand will be useless.

6. Proof of actual damages suffered by the creditor is not necessary in order that the penalty may
be demanded.
a. TRUE. Express provision, Art. 1228.

7. An obligation which is payable upon the death of Mr. X within five years from its execution is a
conditional obligation.
a. TRUE.

8. Solidarity may exist although the creditors and the debtors may not be bound in the same
manner and by the same periods and conditions.
a. TRUE. Art. 1207 provides that “there is a solidary liability only when the obligation
expressly so states, or when the law or the nature of the obligation requires
solidarity.”

Practice Questions and Suggested Answers 4


CIVIL LAW REVIEW II

9. When the fulfillment of the condition depends upon the sole will of the debtor, the obligation
shall be void.
a. FALSE. In resolutory conditional obligations, this does not apply.
b. A resolutory condition is the happening of the condition which extinguishes the
already existing rights and obligations.

10. In conjunctive obligations, the right of choice belongs to the debtor, unless it has been expressly
granted to the creditor.
a. FALSE. A conjunctive obligation is one where the debtor has to perform several
prestations and is only extinguished only by the performance of all of them.

11. It is a concept that derives from the principle that since mutual agreement can create a contract,
__________ by the parties can cause its extinguishment.
a. Mutual disagreement or decent.

12. Where an application for a loan of money is approved, there arises a perfected contract even
before the delivery of the object of the contract.
a. TRUE.

13. If the obligation has been substantially performed, the obligor shall recover as though there had
been a strict and complement fulfillment.
a. FALSE. Performance has to be in good faith. Art. 1234 provides: If the obligation has
been substantially performed ŝŶŐŽŽĚĨĂŝƚŚ, the obligor may recover as though there
had been a strict and complete fulfillment, less damages suffered by the obligee.

14. When the obligee accepts the performance, knowing its incompleteness or irregularity, the
obligation is deemed fully complied with.
a. FALSE. The acceptance must be without expression of any protest or objection. Art.
1235.

15. Payment by a judgment debtor of the wrong party does not extinguish judgment debt.
a. FALSE. This is only the general rule. The following are its exceptions:
i. Art. 1241
ii. Art. 1242
iii. Payment was delivered to the original creditor other than the new creditor.

16. Obligations requiring payment in foreign currency must be discharged in Philippine currency as
provided by RA No. 8183.
a. FALSE. RA 8183 allows the payment in different currency. In the absence of
agreement, payment should be made in Philippine Peso. What the law specifically
prohibits is the payment in currency other than legal tender. (Ponce v. CA)
b. RA 529

Practice Questions and Suggested Answers 5


CIVIL LAW REVIEW II

17. An agreement to pay a promissory note in dollars, while null and void under RA No. 529, does
not defeat a creditor’s claim.
a. TRUE. While an agreement to pay in foreign currency is declared null and void and of
no effect, the law does not defeat a creditor’s claim for payment because what the
law specifically prohibits is the payment in currency other than legal tender. (Ponce v.
CA)

18. A __________ implies that the check is drawn upon sufficient funds in the hands of the drawee,
that they have been set apart for its satisfaction and that they shall be so applied whenever the
check is presented for payment.
a. Certification.

19. The delivery of the promissory notes payable to order, or bills of exchange or other mercantile
documents produces the effect of payment.
a. FALSE. Under Art. 1249, delivery of such documents shall only produce the effect of
payment only when they have been cashed, or when thorough the fault of the
creditor, they have been impaired.

20. The creditor cannot be compelled to partially receive the prestation in which the obligation
consists. Neither may the debtor be required to make partial payments.
a. FALSE. As a general rule, the creditor may not be compelled to accept partial
fulfillment. The following are exceptions to this rule: (1) substantial compliance in
good faith, as provided under Art. 1234; (2) waiver, as provided under Art. 1235; or (3)
application of payments if the debts are equally onerous, as provided in Art. 1254(2).

21. The fact that the debtor had sufficient available funds on or before the grace period for the
payment of its obligation does not constitute proof of tender of payment.
a. TRUE. Tender of payment presupposes not only that the obligor is able, but more so in
the act of performing his obligation.

22. A certified personal check is not legal tender nor the currency stipulated and therefore cannot
constitute valid tender of payment.
a. FALSE. "Though a check is not legal tender, and a creditor may validly refuse to accept
it if tendered as payment, one who in fact accepted a fully funded check after the
debtor’s manifestation that it had been given to settle an obligation is estopped from
later on denouncing the efficacy of such tender of payment." (Far East Bank v. Diaz
Realty, August 23, 2001.)

23. The rule that in case an extraordinary inflation or deflation of the currency stipulated should
supervene, the value of the currency at the time of the establishment of the obligation shall be
the basis of payment is an applicable to obligations arising from tort.

Practice Questions and Suggested Answers 6


CIVIL LAW REVIEW II

a. FALSE. From the employment of the words “extraordinary inflation or deflation of the
currency stipulated”, it can be seen that the same envisages contractual obligations
where a specific currency is selected by the parties as the medium of payment. Thus,
this is inapplicable to obligations arising from torts and not from contracts. (Velasco v.
MERALCO)

24. Under the principle of dation in payment, the mere delivery of the mortgaged motor vehicle by
the mortgagor results in the transfer of ownership to the mortgagee even without the consent
of the latter.
a. FALSE.

25. There can be no consignation when there is no obligation to be extinguished.


a. TRUE. Consignation is the act of depositing the thing due with the court or judicial
authorities whenever the creditor cannot accept or refuses to accept payment and it
generally requires a prior tender of payment. (Likako v. Tedoro)
b. In order that consignation may be effective, the debtor must show that there was a
debt due. (De Leon v. Santiago Syjuco, Inc.)

26. __________ is the act of depositing the thing due with the court or judicial authorities whenever
the creditor cannot accept or refuses to accept payment.
a. Consignation.

27. Tender of payment may be extrajudicial.


a. FALSE. According to Jurado, tender of payment is, by nature, always extrajudicial.

28. Consignation of the amount required is not necessary to preserve the right to redeem.
a. TRUE. The right to redeem is a right, not an obligation. Thus, consignation of the
amount required is not necessary to preserve the right to redeem. (Immaculata v.
Navarro)

29. Difficulty of service authorizes release of obligor but does not authorize courts to modify or
revise the contract between the parties.
a. TRUE. Difficulty of service authorizes the release of the obligor but does not allow the
courts to remake, modify or revise the contract stipulated with the force of law, so as
to substitute its own terms for those covenanted by the parties themselves. (Occena
v. Jabson)

30. Performance is not excused by the fact that the contract turns out to be hard and
improvided(?), unprofitable or unexpectedly burdensome.
a. TRUE.

31. Consent is an essential requisite of obligation.


Practice Questions and Suggested Answers 7
CIVIL LAW REVIEW II

a. FALSE. It is not one of the essential elements because there can be an obligation even
without the consent of one of the parties.

32. A thing is indeterminate if it is not physically segregated from all others of the same class.
a. FALSE. This statement is not absolute. A thing may still be determinate even if not
physically segregated, as when the object is particularly designated.

33. In obligations with a term, the obligation arises upon the arrival of the period.
a. FALSE. The term or period has no effect upon the existence of the obligation, but only
their demandability or performance.

34. In obligations with a penal clause, the debtor may exempt himself from the performance of the
obligation by paying the penalty.
a. TRUE. The debtor cannot pay the penalty to relieve himself of the principal obligation,
unless that right is expressly granted to him. The payment of the penalty in lieu of the
principal obligation can be made only by express stipulation.

35. The insolvency of one debtor will increase the liability of his co-debtors.
a. FALSE. The insolvency of a debtor does not increase the responsibility of his co-
debtors, nor does it authorize a creditor to demand anything from his co-creditors.

36. The indivisibility of an obligation necessarily gives rise to solidarity.


a. TRUE. Express provision.

37. Whenever a period is designated in an obligation, it is for the benefit of both the creditor and
debtor.
a. FALSE. The rule under Art. 1196 is that the period is presumed to have been
established for the benefit of both creditor and debtor, unless from the same or other
circumstances it should appear that the period has been established in favor of one or
the other.

38. When the fulfillment of the obligation is dependent upon the sole will of the debtor, the
conditional obligation shall be void.
a. FALSE. Art. 1182 provides that when the fulfillment of the condition, not the
obligation, is dependent upon the sole will of the debtor, the conditional obligation
shall be void.
i. Even if condition, if condition is resolutory, obligation is not void.

39. In facultative obligations, the right of choice belongs to the debtor unless it has been expressly
granted to the creditor.

Practice Questions and Suggested Answers 8


CIVIL LAW REVIEW II

a. FALSE. The power of the debtor to make substitution is absolute.

40. Consignation, to be valid, generally requires a prior tender of payment.


a. TRUE. As a rule tender of payment is not necessary for a valid consignation; but it is
necessary if the consignation was made when the creditor to whom tender of
payment was made refused to accept it.

41. Consignation of the amount required is not necessary to preserve the right to redeem.
a. TRUE. Since the right to redeem is a right, not an obligation, consignation of the
amount required is not necessary to preserve the right to redeem. (Immaculata v.
Navarro)

42. Difficulty of service authorizes release of obligor but does not authorize courts to modify or
revise he contract between the parties.
a. TRUE. Difficulty of service authorizes the release of the obligor but does not allow the
courts to remake, modify or revise the contract stipulated with the force of law, so as
to substitute its own terms for those covenanted by the parties themselves. (Occena
v. Jabson)

43. It is a concept that derives from the principle that since mutual agreement can create a contract,
mutual disagreement can cause its extinguishment.
a. TRUE.

44. Where an application for a loan of money was approved, there arises a perfected contract
before the delivery of the object of the contract.
a. TRUE.

45. If the obligation has been substantially performed, the obligor shall recover as though there had
been a strict and complete fulfillment.
a. FALSE. Performance has to be in good faith and less damages suffered by the obligee.
Art. 1234 provides: If the obligation has been substantially performed in good faith,
the obligor may recover as though there had been a strict and complete fulfillment,
less damages suffered by the obligee.

46. When the obligee accepts the performance knowing its incompleteness or irregularity, the
obligation is deemed fully complied with.
a. FALSE. The acceptance has to be without expressing any protest or objection.

47. Payment by a judgment debtor to the wrong party does not extinguish judgment debt.
a. FALSE. This is only the general rule. The following are its exceptions:
i. Art. 1241
ii. Art. 1242
Practice Questions and Suggested Answers 9
CIVIL LAW REVIEW II

b. Payment was delivered to the original creditor other than the new creditor.

48. Obligations requiring payment in foreign currency must be discharged in Philippine currency as
provided by RA 8183.
a. FALSE. RA 8183 allows the payment in different currency. In the absence of
agreement, payment should be made in Philippine Peso. What the law specifically
prohibits is the payment in currency other than legal tender. (Ponce v. CA)
b. RA 529

49. An agreement to pay a promissory note in dollars, while null and void under RA 529 does not
defeat a creditor’s claim.
a. TRUE. While an agreement to pay in foreign currency is declared null and void and of
no effect, the law does not defeat a creditor’s claim for payment because what the
law specifically prohibits is the payment in currency other than legal tender. (Ponce v.
CA)

50. Consignation presupposes not only that the obligor is able, ready, and willing to pay, but mores
so in the act of performing his obligation.
a. TRUE.

51. “A proof that an act could have been done is no proof that it was actually done” supports the
concept known as tender of payment.
a. FALSE. Tender of payment presupposes not only that the obligor is able, but more so
in the act of performing his obligation.

52. A certified personal check is not the legal tender nor the currency contemplated, therefore
cannot constitute valid tender of payment.
a. FALSE. "Though a check is not legal tender, and a creditor may validly refuse to accept
it if tendered as payment, one who in fact accepted a fully funded check after the
debtor’s manifestation that it had been given to settle an obligation is estopped from
later on denouncing the efficacy of such tender of payment." (Far East Bank v. Diaz
Realty, August 23, 2001)

53. The rule that in case of extraordinary inflation or deflation of the currency stipulated should
supervene, the value of the currency at the time of the establishment of the obligation shall be
the basis of payment of obligations arising from law.
a. FALSE. From the employment of the words “extraordinary inflation or deflation of the
currency stipulated”, it can be seen that the same envisages contractual obligations
where a specific currency is selected by the parties as the medium of payment. Thus,
this is inapplicable to obligations arising from tots and not from contracts. (Velasco v.
MERALCO)

Practice Questions and Suggested Answers 10


CIVIL LAW REVIEW II

54. Under the principle of dation in payment, what is transferred is merely possession of the
property. (ownership)
a. FALSE. Dation in payment is the delivery and transmission of ownership of a thing by
the debtor to the creditor as an accepted equivalent of the performance of the
obligation. (Filinvest v. Phil. Acetylene)

55. There can be no dation in payment when there is no obligation extinguished.


a. TRUE. Dation in payment is the delivery and transmission of ownership of a thing by
the debtor to the creditor as an accepted equivalent of the performance of the
obligation. (Filinvest v. Phil. Acetylene)

56. Tender of payment may be extrajudicial.


a. FALSE.

57. Consignation is necessarily judicial.


a. TRUE. Art. 1258, “deposit to judicial authorities.”
b. It is judicial for it requires the filing of a complaint in court.

58. Demand is necessary to make an obligation due.


a. FALSE. Demand is not necessary to make an obligation due but to set the other party
in delay and to effectively toll applicable prescriptive periods.
b. As to when an obligation becomes due is not dependent on the existence of demand
in a pure obligation. Every obligation the performance of which does not depend on a
future or uncertain event, or a past event unknown to the parties, is demandable at
once. The same is true in obligations with resolutory condition or a resolutory period,
without prejudice to the effects of the happening of the condition or the arrival of the
period.
c. Also, in a reciprocal and simultaneous obligation, from the moment one of the parties
fulfil his obligation, the other party must be ready to comply with what is incumbent
upon him, or else delay will set in without the necessity of a demand.
i. However, in a reciprocal obligation which is not simultaneous, demand is
generally necessary, again not to make the obligation due but for delay to set
in upon the party who is not able to comply in the proper manner with what is
incumbent upon him.
d. Further, demand is made only upon the obligation becoming due. A demand made
upon an obligation that is not yet due and demandable will not set the other party in
delay. Thus, demand is futile.
e. There are other instances when demand is not necessary as to when the obligation
becomes due, as in:
i. when the law or the obligation expressly so declares, as for example, in a pure
obligation and those subject to a resolutory condition or period, which are due
and demandable at once;
Practice Questions and Suggested Answers 11
CIVIL LAW REVIEW II

ii. when from the nature and circumstances of the obligation it appears that the
designation of the time of fulfilment is the controlling motive of the
establishment of the contract; and
iii. when demand would be useless, as when the obligor has rendered it beyond
his power to perform.

59. The arrival of a period or the happening of a condition gives rise to an obligation.
a. FALSE. This statement is true only as regards obligations subject to a suspensive
condition or suspensive period, wherein the efficacy of the obligation is suspended
until the happening of the suspensive condition or the arrival of the suspensive
period.
b. First, there could be an obligation which is not dependent upon any future or
uncertain event. A pure obligation is demandable at once upon constitution. The
same is true in obligations subject to a resolutory condition or resolutory period,
without prejudice to the effects of the happening of the condition or the arrival of the
period, which extinguishes the obligation.

60. No person shall be responsible for those events which could not be foreseen, or which though
foreseen, were inevitable.
a. FALSE. This statement is generally true, but the civil code admits of exceptions.
b. First, in an obligation to give, the statement is true only when what is to be delivered
is a determinate thing. If it was a generic thing, the obligation will not necessarily be
extinguished by the happening of a fortuitous event, because of the principle that
genus never perishes.
c. The exceptions to this rule as provided by the civil code include, when the parties
expressly so stipulates the person to be responsible even for fortuitous events; or
when the law so provides; or when the nature of the obligation requires the
assumption of risks; or when the obligor is in delay upon the happening of the
fortuitous event; or when he has promised to deliver the same thing to two or more
persons who do not have the same interest, then the statement will not necessarily be
true anymore.

61. The value of the currency at the time of the constitution of the obligation shall be the basis of
payment in case of extraordinary inflation or deflation.
a. FALSE. This statement will only be true as regards obligations arising from contracts, in
case an extraordinary inflation or deflation of the currency stipulated should
supervene. The same provision already has an exception, where the parties agree on
the contrary. The provision speaks of parties and of agreement, thereby presupposing
that a contract was entered into between them. In obligations arising from law,
quasi-contracts, delicts and quasi-delicts, the applicable law or statutes should govern.

Practice Questions and Suggested Answers 12


CIVIL LAW REVIEW II

OBLIGATIONS

TRUE OR FALSE

1. Consent is an essential requisite of obligations.


a. FALSE. Not all obligations require consent; only contracts require consent.

2. A thing is indeterminate if it is not physically segregated from all others of the same class.
a. FALSE. This statement is not absolute. A thing may still be determinate even if not
physically segregated, as when the object is particularly designated.

3. In obligations with a term, the obligation arises upon the arrival of the period.
a. FALSE. The term or period has no effect upon the existence of the obligation, but only
their demandability or performance.
b. The effect of arrival of a period only goes into the demandability.

4. In obligations with a penal clause, the debtor cannot exempt himself from the performance of
the obligation by paying the penalty.
a. FALSE. This must be expressly granted to the debtor.

5. In obligations with a penal clause, the penalty shall substitute the indemnity for damages and
the payment of interest.
a. FALSE. The statement is not absolute; an exception is if there was an agreement.

6. The insolvency of one debtor will increase the liability of his co-debtors.
a. FALSE. This does not apply in obligations where the debtors are joint.

7. The indivisibility of an obligation does not necessarily give rise to solidarity.


a. TRUE. Express provision.

8. Whenever a period is designated in an obligation, it is for the benefit of both the creditor and
the debtor.
a. FALSE. This is a disputable presumption.
b. The rule under Art. 1196 is that the period is presumed to have been established for
the benefit of both creditor and debtor, unless from the same or other circumstances
it should appear that the period has been established in favor of one or the other.

9. When the fulfillment of the condition depends upon the sole will of the debtor, the conditional
obligation shall be void.
a. FALSE. This will only apply if the condition is suspensive. If the condition is resolutory,
the obligation shall be deemed valid.

Practice Questions and Suggested Answers 13


CIVIL LAW REVIEW II

10. In facultative obligations, the right of choice belongs to the debtor, unless it has been expressly
granted to the creditor.
a. FALSE. The choice only belongs to the debtor. The power of the debtor to make
substitution in facultative obligations is absolute.

MULTIPLE CHOICE QUESTIONS

1. A, B and C borrowed P36,000 from X. the debtors signed a promissory note on January 10, 2000,
promising to pay the creditor jointly and severally on or before July 10, 2000. How much can X
collect from C?
a. P18,000
b. P6,000
c. P12,000
d. P36,000
i. The debtors are bound solidarily, hence, any one of them may be made to pay
the whole amount of the obligation.
ii. “Jointly and severally” means solidary.

2. On January 1, 1999, A signs a promissory note and binds himself to pay X P100,000 plus 15% per
annum interest on June 30, 1999.
a. Before June 30, 1999, X can demand payment
b. IF on June 30, 1999, A is paying X, X can refuse the payment
c. Because the period is for the benefit of the debtor, A can compel the creditor X to
accept payment any date before June 30, 1999.
d. Because the period is for the benefit of the debtor and the creditor, X can refuse any
rendered payment before June 30, 1999.
i. The law presumes that the period is for the benefit of both debtor and
creditor. In this case, the period allows the debtor to raise money to pay the
obligation, while the creditor has interest.

3. Bertulfo and Claudio promised to deliver a particular car valued at P100,000 to Manuela on or
before September 15, 1999. September 15, 1999 came and upon demand by Manuela for
delivery from Bertulfo and Claudio, Bertulfo was willing to deliver but Claudio refused to deliver.
Thereafter, the car was destroyed. In this case:
a. An action for specific performance will lia against both Bertuflo and Claudio.
b. Both Bertulfo and Claudio shall be liable for P50,000 each with damages.
c. Bertulfo shall be liable for P50,000 without damages and Claudio shall be liable for
P50,000 and damages.
i. The obligation is joint and indivisible but it has become impossible to perform
due to the fault of one of the debtors. Thus, the obligation is converted to a
monetary obligation with damages for the debtor at fault.
d. An action for specific performance will lie against Claudio only because of his refusal.

Practice Questions and Suggested Answers 14


CIVIL LAW REVIEW II

4. In an obligation where only one prestation has been agreed upon, but to extinguish the
obligation, the debtor is allowed to render another prestation, the obligation is:
a. Disjunctive obligation
b. Facultative obligation
c. Obligation with penal clause
d. Alternative obligation
e. Joint obligation

5. An obligation ceases to be alternative and becomes a simple obligation:


a. When the debtor has already made a choice
b. When the creditor has already made a choice
c. When the choice of the debtor is consented to by the creditor
d. When the choice of the creditor is consented to by the debtor
e. None of the above
i. Correct answer is when the choice has been communicated to the other party.

6. It refers to a joint obligation:


a. One in which each debtor is liable for the entire obligation, and each creditor is entitled
to demand the whole obligation.
b. One in which either one of the parties is indispensable and the other is not necessary.
c. One in which the obligation of one is a resolutory condition of the obligation of the
other, the non0fulfillment of which entitles the other party to rescind the contract.
d. One in which each of the debtors is liable only for a proportionate part of the debt and
each creditor is entitled only for a proportionate part of the credit.

7. A and B are solidary debtor of X and Y, solidary creditors to the amount of P4,000. On the due
date, X renounced gratuitously in favor of A the entire obligation. Which of the following is
correct?
a. B shall give A P2,000
b. Y can still collect from B P2,000
c. Any of the two
d. None of the above
i. Renunciation of the entire obligation shall result in its extinguishment in
SOLIDARY OBLIGATIONS.

8. The creditor has the right to the fruits of the thing from:
a. The time the thing is delivered
b. The time the obligation to deliver the thing arises
c. The time the contract is perfected
d. The time the fruits are delivered

Practice Questions and Suggested Answers 15


CIVIL LAW REVIEW II

9. The issue of who has the right to choose may not occur in:
a. Conjunctive obligations
b. Facultative obligations
c. Alternative obligations
d. Both A and B
i. In conjunctive obligations, there is no issue regarding choice because all of the
prestations must be delivered for extinguishment.
ii. In facultative obligations, there is no issue regarding choice because the
debtor’s right of choice is absolute.
e. A, B and C
f. None of the above

10. D obliged himself to give C, object No. 1 valued P15,000; or object No. 2 valued P10,000; or
object 3 valued P5,000. All the objects were lost due to D’s fault in the order as stated.
a. D’s obligation is extinguished.
b. D’s obligation is to pay the value of object No.1 plus damages
c. C’s right to demand the value of any of the objects plus damages.
d. D’s obligation is to pay the value of object No. 3 plus damages.
i. In alternative obligations, the right of choice is presumed to belong to the
debtor.

Practice Questions and Suggested Answers 16


CIVIL LAW REVIEW II

KINDS OF OBLIGATIONS

TRUE OR FALSE

1. Every obligation whose performance does not depend upon a future and uncertain event, or
upon a past event unknown to the parties, is demandable at once.
a. FALSE. Art. 1179 – Every obligation whose performance does not depend upon a
future OR uncertain event, or upon a past event unknown to the parties, is
demandable at once.”
b. The statement is incomplete because it does not take into consideration obligations
with a period which depends upon a future and CERTAIN event.

2. Conditions which are contrary to public policy shall annul the obligation which depends upon
them.
a. TRUE. Art. 1183 – Impossible conditions, those contrary to good customs or public
policy, and those prohibited by law shall annul the obligation which depends upon
them.
b. While Art 1183 also talks about the obligation not being annulled if it is divisible, the
statement does not go into the divisibility or indivisibility of the same.

3. The condition shall be deemed fulfilled when the obligee voluntarily prevents its fulfillment.
a. FALSE. Art. 1186 – The condition shall be deemed fulfilled when the OBLIGOR
voluntarily prevents its fulfillment.
b. It is absurd for the obligee (creditor) to voluntarily prevent the fulfillment of the
condition. K

4. Obligations for whose fulfillment a day certain has been fixed, shall be demandable only when
that day comes.
a. TRUE. Art. 1193 – Obligations for whose fulfillment a day certain has been fixed, shall
be demandable at once. Obligations with a resolutory period take effect at once, but
terminate upon the arrival of the day certain.
b. The statement is absolutely true because it only pertains to obligations with a
suspensive period.

5. Whenever in an obligation a period is designated, it is deemed to have been established for the
benefit of both the creditor and the debtor.
a. FALSE.
b. Art. 1196 – Whenever in an obligation, a period is designated, it is WZ^hD to have
been established for the benefit of both the creditor and debtor, ƵŶůĞƐƐĨƌŽŵƚŚĞƚĞŶŽƌ
ŽĨ ƚŚĞ ƐĂŵĞ Žƌ ŽƚŚĞƌ ĐŝƌĐƵŵƐƚĂŶĐĞƐ͕ ŝƚ ƐŚŽƵůĚ ĂƉƉĞĂƌ ƚŚĂƚ ƚŚĞ ƉĞƌŝŽĚ ŚĂƐ ďĞĞŶ
ĞƐƚĂďůŝƐŚĞĚŝŶĨĂǀŽƌŽĨŽŶĞŽƌŽĨƚŚĞŽƚŚĞƌ͘

6. If the obligation does not fix a period, but from its nature and the circumstances it can be
inferred that a period was intended, the courts may fix the duration thereof.
a. TRUE. Art. 1197

Practice Questions and Suggested Answers 17


CIVIL LAW REVIEW II

7. The debtor shall lose every right to make use of the period when he does not furnish to the
creditor the guaranties or securities demanded by the latter after the obligation has been
contracted.
a. FALSE. Art. 1198(2) – The debtor shall lose every right to make use of the period when
he does not furnish to the creditor the guarantees or securities WHICH HE HAS
PROMISED.
b. Statement will only be true if debtor promised to furnish guaranties or securities.

8. A person alternatively bound by different prestations shall completely perform all of them.
a. FALSE. Art. 1199 – A person alternatively bound by different prestations shall
completely perform ONE of them.
b. A conjunctive obligation is one where the debtor has to perform several prestations; it
is extinguished only by the performance of ALL of them.

9. If through the debtor’s acts the creditor cannot make a choice according to the terms of the
obligation, the latter may rescind the contract with damages.
a. FALSE. Art. 1203 – If through the CREDITOR’S acts the debtor cannot make a choice
according to the terms of the obligation, the latter may rescind the contract with
damages.

10. The loss or deterioration of the thing intended as a substitute, through the negligence of the
obligor, does not render him liable.
a. TRUE. Art. 1206 – The loss or deterioration of the thing intended as a substitute,
through the negligence of the obligor, does not render him liable. But once the
substitution has been made, the obligor is liable for the loss of the substitute on
account of his delay, negligence, or fraud.
b. The statement is true because the first part of the provision used the word “intended”
which presupposes that the substitution HAS NOT YET BEEN MADE. The second
statement is not an exception to the first one because both contemplate different
scenarios.

MULTIPLE CHOICE QUESTIONS

1. D obliged himself to allow C the use of his car until C passes the CPA Examination. This is an
example of:
a. a pure obligation
b. an obligation with a suspensive condition
c. an obligation with a resolutory condition
d. an obligation with a suspensive period
e. an obligation with a resolutory period

· It is resolutory because the obligation is demandable at once. The statement also used the
word “until” which is an indicator that such is resolutory.
· It is a condition, not a period, because passing the CPA examination is a future and
UNCERTAIN event.

2. When the debtor binds himself to pay when his means permit him to do so, the obligation is:
a. a pure obligation

Practice Questions and Suggested Answers 18


CIVIL LAW REVIEW II

b. an obligation with a suspensive condition


c. an obligation with a resolutory condition
d. an obligation with a suspensive period
e. an obligation with a resolutory period

· Art. 1180 – When the debtor binds himself to pay when his means permit him to do so,
the obligation shall be deemed to be one with a period, subject to the provisions of Article
1197.
· The period is suspensive because the obligation will only be demandable upon the arrival
of the day when the debtor’s means permit him to do so.

3. The following obligations are immediately demandable, EXCEPT:


a. a pure obligation
b. obligation with a resolutory condition
c. obligation with a resolutory period
d. all of the above
e. none of the above

· Pure obligations are demandable at once. (Art. 1179)


· Obligations with a resolutory condition or period are also demandable at once. (Art. 1179
and Art. 1193)

4. In an obligation where only one prestation has been agreed upon, but to extinguish the
obligation the debtor is allowed to render another prestation, the obligation is a:
a. disjunctive obligation
b. obligation with a penal clause
c. alternative obligation
d. joint obligation
e. none of the above

· The statement refers to a facultative obligation.

5. An obligation ceases to be alternative and becomes a simple obligation:


a. when the debtor has already made a choice
b. when the creditor has already made a choice
c. when choice of the debtor is consented to by the creditor
d. when choice of the creditor is consented to by the debtor
e. none of the above

· Art. 1205 – The obligation shall cease to be alternative from the day when the selection
has been communicated to the debtor (or creditor, depending on who has the right to
choose).

6. “I promise to give A a car upon A’s passing the bar examinations.” The condition in this
obligation is:
a. resolutory
b. casual

Practice Questions and Suggested Answers 19


CIVIL LAW REVIEW II

c. possible
d. all of the above
e. none of the above

· A possible is one which is capable of fulfillment according to nature, law, public policy, or
good customs.
· It is not resolutory because it is not a certain event.
· The worst answer is CASUAL, because a casual condition is that which depends
EXCLUSIVELY upon chance or other factors, and NOT UPON THE WILL OF THE
CONTRACTING PARTIES. J

7. Which of the following conditional obligations is void?


a. When the happening of the suspensive condition is dependent upon the sole will of the
creditor.
b. When the happening of the suspensive condition is dependent upon the sole will of
the debtor.
c. When the happening of the suspensive condition is dependent upon chance.
d. When the happening of the resolutory condition is dependent upon the sole will of the
creditor.
e. When the happening of the resolutory condition is dependent upon the sole will of the
debtor.

· Art. 1182 – When the fulfillment of the condition depends upon the sole will of the
debtor, the conditional obligation shall be void.
· This applies only to conditions which are suspensive. It does not cover resolutory
conditions.

8. On January 1, 1999, A signed a promissory note binding himself to pay X P100,000 on or before
June 30, 1999.
a. Before June 30, 1999, A cannot be compelled to pay.
b. Before June 30, 1999, X can validly refuse an offer to pay.
c. Before June 30, 1999 while A cannot be compelled to pay, X also cannot be compelled
to accept payment.
d. Before June 30, 1999 while A can be compelled to pay, X cannot be compelled to accept
payment.
e. A, B, and C are correct.

· The period designated is for the benefit of the debtor only. Thus, he can oppose a
premature demand for payment, but may validly pay at any time before the period
expires.
· The phrase “on or before” is an indicator that the period has been established for the
benefit of the debtor.

9. The issue of who has the right to choose may NOT occur in:
a. conjunctive obligations
b. facultative obligations
c. alternative obligations

Practice Questions and Suggested Answers 20


CIVIL LAW REVIEW II

d. both a and b
e. a, b, and c

· In conjuctive obligations, the right to choose is a non-issue because all the prestations
have to be performed anyway.
· In facultative obligations, the right to choose is ALWAYS vested in the debtor while in
alternative obligations, the right may pertain even to the creditor or to a third person
when it has been expressly granted to either of them.

10. D obliged himself to give C, object no. 1 valued at P15,000; or object no. 2 valued at P10,000; or
object no. 3 valued at P5,000. All the objects were lost due to D’s fault in the order as stated. If C
has the right to choose:
a. D’s obligation is extinguished.
b. D’s obligation is to pay the value of object no. 1 plus damages.
c. C’s right to demand the value of any of the objects plus damages.
d. D’s obligation is to pay the value of object no. 3 plus damages.

· Art. 1205(3) – When the choice has been expressly given to the creditor, the obligation
shall cease to be alternative from the day when the selection has been communicated to
the debtor. … Until then, if all things are lost through the fault of the debtor, the choice by
the creditor shall fall upon the price of any one of them, also with indemnity for damages.

Practice Questions and Suggested Answers 21


CIVIL LAW REVIEW II

JOINT AND SOLIDARY OBLIGATIONS (computation)

1. in a joint obligation with V, W, X, Y and Z as debtors and A and B as creditors in the amount of
P900,000:
a. How much can A validly demand from V?
i. P90,000
ii. Pursuant to Art. 1208, there are a total of 10 obligations (5 debtors; 2
creditors)
b. How much can V recover from W if he paid the amount that A can compel him to pay?
i. None. V only paid his own share in the debt.
c. How much should A give to B if V paid him the amount that he can compel V to pay?
i. None. A only collected his share in the credit.

2. In a solidary obligation with V, W, X, Y and Z as debtors and A and B as creditors in the amount
of P900,000:
a. How much can A validly demand from X?
i. P900,000
ii. In solidary obligations, any one of the debtors may be compelled to pay for
the whole amount of the debt.
b. How much can X recover from W if he paid the amount that A can compel him to pay?
i. P180,000
ii. Unless there is a stipulation, debtors in a solidary obligation share equally in
paying the entire obligation.
c. How much should A give to B if X paid him the amount that he can compel X to pay?
i. P450,000
ii. Unless there is a stipulation, creditors in a solidary obligation share equally in
the credit.

3. In a passive solidarity with V, W, X, Y and Z as debtors and A and B as creditors in the amount of
P900,000:
a. How much can A validly demand from V?
i. P450,000
ii. In passive solidarity, solidarity exists among the debtors. A creditor can only
demand from any of the debtors his share in the credit.
b. How much can V recover from Z if he paid the amount that A can compel him to pay?
i. P90,000
c. How much should A give to B if V paid him the amount that he can compel V to pay?
i. None. A only collected his share in the credit.

4. In a solidary obligation with V, W, X, Y and Z as debtors and A and B as creditors in the amount
of P900,000 and V is insolvent:
a. How much can A validly demand from X?
Practice Questions and Suggested Answers 22
CIVIL LAW REVIEW II

i. P900,000
b. How much can X recover from Y if he paid the amount that A can compel him to pay?
i. P225,000
ii. Since the obligation is solidary, the insolvency of one of the debtors increases
the liability of the co-debtors.
c. How much should A give to B if X paid him the amount that he can compel X to pay?
i. P450,000

5. In a joint obligation with V, W, X, Y and Z as debtors and A and B as creditors in the amount of
P900,000 where V is insolvent:
a. How much can A validly demand X?
i. P90,000
ii. In joint obligations, the insolvency of one debtor does not increase the liability
of his co-debtors.
b. How much can X recover from Y if he paid the amount that A can compel him to pay?
i. None. X only paid for his share in the debt.
c. How much should A give to B if X paid him the amount that he can compel X to pay?
i. None. A only collected his share in credit.

6. In a solidary obligation with V, W, X, Y and Z as debtors and A and B as creditors in the amount
of P900,000, where V is insolvent and A remitted the share of Z with the consent of B:
a. How much can A validly demand from X?
i. P720,000
ii. The remission of A reduces the amount of the debt by the share of Z
(P180,000).
iii. Since the obligation is solidary, the insolvency of one debtor increases the
liability of the other co-debtors.
b. How much can X recover from Z if he paid the amount that A can compel him to pay?
i. P45,000
ii. Since the obligation is solidary, the insolvency of one debtor increases the
liability of the other co-debtors.
iii. This is Z’s share in covering the amount that V, their co-debtor, was unable to
pay due to his insolvency.
c. How much should A give to B if X paid him the amount that he can compel X to pay?
i. P360,000

7. In a solidary obligation with V, W, X, Y and Z as debtors and A and B as creditors in the amount
of P900,000, how much can A validly demand from W if V was a minor at the time the obligation
was constituted?
a. P720,000
b. Since the obligation is solidary, each debtor shall share equally in the debt, unless
there is a contrary stipulation. Thus, each co-debtor is liable for P180,000.
Practice Questions and Suggested Answers 23
CIVIL LAW REVIEW II

c. Minority of one of the co-debtors is a partial defense of the other co-debtors who are
not minors. Such partial defense may be invoked only insofar as the share of the
minor co-debtor.

8. In a solidary obligation with V, W, X, Y and Z as debtors and A and B as creditors in the amount
of P900,000, where A remitted the share of Z without the consent of B, how much can B validly
demand from X?
a. P720,000
b. Since the remission was without the consent of A, B can demand from him his whole
share of P450,000.
i. Hindi mababawasan ung share ni B because of the remission because he did
not consent to such.

Practice Questions and Suggested Answers 24


CIVIL LAW REVIEW II

MODES OF EXTINGUISHMENT

TRUE OR FALSE

1. Payment made by the debtor to a third person who is not the creditor shall never extinguish the
obligation.
a. FALSE.
b. Art. 1241 – Payment to a third person shall also be valid insofar as it has redounded to the
benefit of the creditor.
c. Art. 1242 – Payment made in good faith to any person in possession of the credit shall
release the debtor.
d. When creditor is incapacitated to receive payment or performance.

2. The delivery to and acceptance by the creditor of a cashier’s check produces the effect of
payment of the debtor’s obligation.
a. FALSE. Art. 1249 – The payment of debts in money shall be made in the currency
stipulated, and if it is not possible to deliver such currency, then in the currency which is
legal tender in the Philippines.
b. The delivery of promissory notes payable to order, or bills of exchange, or other
mercantile documents shall produce the effect of payment ONLY when they have been
cashed, or when through the fault of the creditor, they have been impaired.

3. Payment by cession does not always extinguish the whole obligation.


a. TRUE. Art. 1255 – This cession, unless there is a stipulation to the contrary, shall only
release the debtor from responsibility for the net proceeds of the thing assigned.

4. Legal subrogation requires the consent of the original parties and of the third person.
a. FALSE. Art. 1301 – Conventional subrogation of a third person requires the consent of the
original parties and of the third person.
b. Legal subrogation is subrogation by law. Unless there is a specific law providing for it, its
existence cannot be presumed. The consent of the parties has absolutely nothing to do
with it.

5. In novation, if the new obligation is void, the original one shall subsist, unless the parties
intended that the former relation should be extinguished in any event.
a. TRUE. Art. 1297

6. When the principal obligation is extinguished in consequence of a novation, accessory


obligations subsist.
a. FALSE. Art. 1296 – When the principal obligation is extinguished in consequence of a
novation, accessory obligations MAY subsist only insofar as they may benefit third persons
who did not give their consent.

7. Novation may be made even without the knowledge of the creditor.


a. FALSE. Art. 1293 – Novation may be made even without the knowledge or against the will
of the latter (debtor), but not without the consent of the creditor.
b. Consent of the creditor is ALWAYS necessary in novation.

Practice Questions and Suggested Answers 25


CIVIL LAW REVIEW II

8. Payment shall be made to one of the parties to the constitution of the obligation to extinguish
the obligation.
a. FALSE. Art. 1240 – Payment shall be made to the person in whose favor the obligation has
been constituted, or his successor in interest, or any person authorized to receive it.

9. Payment made in good faith to any person in possession of the evidence of credit shall release
the debtor.
a. FALSE. Art. 1242 – Payment made in good faith to any person in POSSESSION OF THE
CREDIT shall release the debtor.
b. Possession of credit IS DIFFERENT with possession of evidence of credit

10. Dation in payment is governed by the law of sales.


a. FALSE. Art. 1245 – Dation in payment, whereby the property is alienated to the creditor in
satisfaction of a DEBT IN MONEY, shall be governed by the law of sales.
b. If debt in money – law on sales; if debt is non-monetary – law on novation shall apply.

11. Legal compensation shall not be proper when one of the debts arises from a depositum.
a. TRUE. Art. 1287 – Compensation shall not be proper when one of the debts arises from a
depositum or from the obligations of a depositary or of a bailey in commodatum.
b. “Legal compensation is the true kind of compensation.” (URIBE) Thus, this provision
applies to all kinds of compensation, including legal compesation.

12. If a person should have against him several debts which are susceptible of compensation, the
rules on the application of payments shall apply to the order of the compensation.
a. TRUE. Art. 1289.

13. In partial compensation, no obligation is extinguished.


a. FALSE. Art. 1290 – When all the requisites mentioned in Art 1279 are present,
compensation takes place by operation of law, and extinguishes BOTH DEBTS TO THE
CONCURRENT AMOUNT, even though the creditors and debtors are not aware of the
compensation.

14. When one or both debts are rescissible or voidable, they may be compensated against each
other before they are judicially rescinded or avoided.
a. TRUE. Art. 1284.

15. In case an extraordinary inflation or deflation of the currency stipulated should supervene, the
value of the currency at the time of payment shall be the basis of payment.
a. FALSE. Art. 1250 – In case an extraordinary inflation or deflation of the currency stipulated
should supervene, the value of the currency at the time of the ESTABLISHMENT OF THE
OBLIGATION shall be the basis of payment, unless there is an agreement to the contrary.

16. If the debt produces interest, payment of the principal shall not be deemed to have been made
until the interests have been covered.
a. TRUE. Art. 1253.

17. An obligation which consists in the delivery of a determinate thing shall be extinguished if it
should be lost or destroyed due to a fortuitous event.

Practice Questions and Suggested Answers 26


CIVIL LAW REVIEW II

a. TRUE. Art. 1262 – An obligation which consists in the delivery of a determinate thing shall
be extinguished if it should be lost or destroyed without the fault of the debtor, and
before he has incurred in delay.
b. The delay is not a factor to consider in determining whether the obligation has been
extinguished or not. It is only relevant insofar as the remedies of the creditor against the
debtor are concerned.

18. Compensation shall take place in reciprocal obligations.


a. FALSE. Art. 1279 – In order that compensation may be proper, it is necessary:
(2) That both debts consist in a sum of money, or if the things due are
consumable, they be of the same kind, and also of the same quality if the latter
has been stated; …
b. In reciprocal obligations, there are different prestations, one is delivery of a thing and the
other is monetary.

19. The guarantor may not set up compensation as regards what the creditor may owe the principal
debtor
a. FALSE. Art. 1280 – Notwithstanding the provisions of the preceding article, the guarantor
MAY set up compensation as regards what the creditor may owe the principal debtor.

20. The obligation is extinguished from the time the characters of the creditor and debtor are
compensated in the same person.
a. FALSE. Art. 1275 – The obligation is extinguished from the time the characters of the
creditor and debtor are MERGED in the same person.

MULTIPLE CHOICE QUESTIONS

1. In three of the following cases, compensation shall NOT be proper. Which is the exception?
a. commodatum
b. gratuitous support
c. civil liability arising out of criminal offenses
d. bank deposit

2. When two persons in their own right are debtors and creditors of each other, there is:
a. confusion
b. compensation
c. consignation
d. novation

· Art. 1278 – Compensation shall take place when two persons, in their own right, are creditors
and debtors of each other.

3. A owes X P50,000 payable on or before June 30, 1999. S, who is not a party to the contract and
without the consent and against the will of A, paid X the P50,000 on April 1, 1999 when the
prevailing rate of interest was 12% per annum.
a. S can ask reimbursement from A in the amount of P50,000 plus 12% interest from April
1 to June 30, 1999.
b. S can ask reimbursement from A in the amount of P50,000.

Practice Questions and Suggested Answers 27


CIVIL LAW REVIEW II

c. S cannot ask reimbursement from A because the payment by S is without the consent
and against the will of A.
d. S can ask refund from X because the payment by S was against the will of A.

· Art. 1236 – Whoever pays for another may demand from the debtor what he has paid, except
that if he paid without the knowledge or against the will of the debtor, he can recover only
insofar as the payment has been beneficial to the debtor.
· Since the payment was made before June 30, there is no interest due to X.
· Also, because the payment was made without the consent and against the will of A, X can only
ask for reimbursement to the extent that A was benefited.

4. When an obligation is extinguished because of the passage of time, this is:


a. fulfillment of a resolutory condition
b. arrival of a resolutory period
c. novation
d. rescission

· A condition refers to an event, while a term or period refers to an interval of time. Since the
statement referred to the “passage of time,” the better answer is the arrival of a resolutory
period.

5. The following shall produce the effect of payment of debts:


a. delivery of check
b. tender of central bank notes
c. delivery of promissory note
d. dacion en pago

· The delivery of a check or of a promissory note will not produce the effect of payment UNLESS
they have been cashed or through the fault of the creditor, they have been impaired. (Art.
1249)
· The tender of central bank notes (money J) will also not produce the effect of payment if the
tender was not accepted.

6. Which of the following will NOT necessarily extinguish an obligation?


a. condonation
b. novation
c. death of the obligor
d. mutual dissent

· The death of the obligor will only extinguish an obligation if it is PURELY PERSONAL.

7. If a person pays the creditor without the knowledge or against the will of the debtor, the
following are true, EXCEPT:
a. He can recover only insofar as the payment has been beneficial to the debtor.
b. He cannot recover the amount from the creditor as payment of what is not due.
c. His remedy is only a simple personal action for reimbursement.
d. He can be subrogated to the rights of the creditor if the latter agrees.

Practice Questions and Suggested Answers 28


CIVIL LAW REVIEW II

· When payment by a third person is made without the knowledge of the debtor or against his
will, there will be no right of subrogation.

8. Payment made to a third person will NOT be valid if:


a. The payment redounded to the benefit of the creditor.
b. The payment was made to a person in possession of evidence of credit.
c. The payment was given to the original creditor after an assignment of the credit without
notice to the debtor.
d. The payment was made to an agent of the creditor who was authorized to receive
payment.

· Art. 1242

9. Which of the following will always extinguish an obligation?


a. tender of payment
b. insolvency of the debtor
c. death of the obligor
d. none of the above

· Tender of payment without acceptance from the creditor will not extinguish the obligation.
· The fact of insolvency of the debtor will also not extinguish the obligation. It is only upon the
declaration of the courts that the insolvency of the debtor may extinguish the obligation.
· The death of the obligor only extinguishes obligations which are purely personal.

10. X owes Y P40,000. With the consent of both parties, Z pays Y P20,000 which makes Y and Z
creditors of X to the amount of P20,000 each. X becomes bankrupt and has only P20,000 assets
left.
a. Z should get the P20,000.
b. Y and Z should divide the P20,000 equally.
c. X may choose whom to pay.
d. Y should get the P20,000.

· The original creditor is preferred over the new creditor in case the assets of the debtor is
insufficient to pay the debt.

Practice Questions and Suggested Answers 29


CIVIL LAW REVIEW II

CONTRACTS

TRUE OR FALSE

1. Contracts must bind both parties, its validity or termination cannot be left to the will of one of
them.
a. FALSE.
b. Its validity or ĐŽŵƉůŝĂŶĐĞ͕ ŶŽƚƚĞƌŵŝŶĂƚŝŽŶ, cannot be left to the will of one of them.
(Art. 1308)

2. Unenforceable contracts are susceptible of ratification.


a. TRUE.
b. Unenforceable, unless they are ratified. (Art. 1403)

3. Contracts are perfected by mere consent.


a. FALSE.
b. This statement is only the general rule. An exception – real contracts such as deposit,
pledge and commodatum, are not perfected until the delivery of the object of the
obligation. (Art. 1316)

4. Ratification requires conformity of the contracting parties.


a. FALSE.
b. Ratification does not require the conformity of the contracting party ǁŚŽŚĂƐŶŽƌŝŐŚƚ
ƚŽďƌŝŶŐƚŚĞĂĐƚŝŽŶĨŽƌĂŶŶƵůŵĞŶƚ͘(Art. 1395)

5. Mistake as to the qualifications of one of the parties vitiates consent.


a. FALSE.
b. It will only vitiate consent when such qualification or identity have been the principal
cause of the contract. (Art. 1331)

6. Contracts take effect between their parties, assigns and heirs.


a. FALSE.
b. The statement is not absolute as when the rights and obligations arising from the
contract are not transmissible by their nature, or by stipulation, or by provision of law.
(Art. 1311)

7. A mere expression of an opinion does not signify fraud.


a. FALSE.
b. This is not absolute. It will only signify fraud when it is made by an expert and the
other party has relied on the expert’s special knowledge. (Art. 1341)

8. Incidental fraud invalidates a contract.


a. FALSE.
b. In order that fraud may make a contract voidable, it should be serious and should not
have been employed by both contracting parties. Incidental fraud only obligates the
person employing it to pay damages. (Art. 1344)

Practice Questions and Suggested Answers 30


CIVIL LAW REVIEW II

c. Dolo incident is fraud that does not have such a decisive influence and by itself cannot
cause the giving of consent; it refers only to some particular or accident of the
obligation.

9. In contracts of pure beneficence, the cause is the liberality of the benefactor.


a. TRUE.

10. The contract is void if the cause is not stated in the contract.
a. FALSE.
b. Although the cause is not stated, it is presumed that it exists and is lawful, unless the
debtor proves the contrary.

11. The contracting parties may establish such stipulations, clauses, terms and conditions as they
may deem convenient.
a. TRUE.

12. Acceptance by letter or telegram binds the offeror from the time the letter or telegram is sent.
a. FALSE.
b. It binds the offerer from the time it came to his knowledge. (Art. 1319(2))

13. An offer made through the agent is accepted from the time acceptance is communicated to the
agent.
a. TRUE.

14. There is violence when in order to wrest consent, serious or irresistible force is employed.
a. TRUE.

15. Failure to disclose facts constitutes fraud.


a. FALSE.
b. Failure to disclose facts will constitute fraud only when there is a duty to reveal them,
as when parties are bound by confidential relations. (Art. 1339)

16. Real contracts, such as mutuum, pledge and guaranty are not perfected until the delivery of the
object of the obligation.
a. FALSE.
b. Real contracts such as ĚĞƉŽƐŝƚ͕ƉůĞĚŐĞĂŶĚĐŽŵŵŽĚĂƚƵŵ͕ŶŽƚŵƵƚƵƵŵĂŶĚŐƵĂƌĂŶƚLJ,
are not perfected until the delivery of the object of the obligation. (Art. 1316)

17. Contracts of pure beneficence are contracts without cause.


a. FALSE.
b. In contracts of pure beneficence, the cause is the mere liberality of the benefactor.
(Art. 1350)

18. If the cause is not stated in the contract, it exists and is lawful.
a. FALSE.
b. It is presumed to be valid and existing, unless otherwise proven by the debtor.

Practice Questions and Suggested Answers 31


CIVIL LAW REVIEW II

19. A contract may be voidable even though there may have been no damage to the contracting
parties.
a. TRUE.

20. The action or defense for the declaration of the inexistence of a contract prescribes in 10 years.
a. FALSE.
b. The action or defense for the declaration of the inexistence of a contract does not
prescribe. (Art. 1410)

21. Acts and contracts which have for their objects the creation transmission, modification or
extinguishment of real rights over immovable property must be in a public instrument to be
valid and binding.
a. FALSE. NŽƚĨŽƌǀĂůŝĚŝƚLJďƵƚĨŽƌŐƌĞĂƚĞƌĞĨĨŝĐĂĐLJ͘Art. 1358 does not require such form
in order to validate the act or contract, but only to insure its efficacy. (Doliendo v.
Depino)

22. Contract of lease for 2 years is unenforceable if not in writing.


a. FALSE. An agreement for the leasing for a longer period than one year of ƌĞĂůƉƌŽƉĞƌƚLJ
ŽƌŽĨĂŶŝŶƚĞƌĞƐƚƚŚĞƌĞŝŶ, should be in writing, otherwise it will be unenforceable. (Art.
1403(2e)
b. KŶůLJŝĨůĞĂƐĞŽĨƌĞĂůƉƌŽƉĞƌƚLJ͘

23. An action for rescission of a contract entered into to defraud a creditor will prosper 4 years from
actual discovery of fraud.
a. FALSE. “From time of registration”.

24. Undue influence shall annul the obligation, although it may have been employed by a third
person who did not take part in the contract.
a. TRUE.

25. Rescission shall be only to the extent necessary to cover the damage caused by a contract in
fraud of a creditor.
a. TRUE.

26. A conveyance is presumed to be in fraud of creditor if made by a debtor after a suit is filed
against him while it is pending.
a. FALSE. Alienations by onerous title are also presumed fraudulent when made by
persons against whom some judgment has been issued. (Art. 1387)
b. ĐŽŶǀĞLJĂŶĐĞŵƵƐƚďĞŽŶĞƌŽƵƐƚŝƚůĞ

27. A contract which is void on its face cannot be the subject of reformation.
a. FALSE. The action for reformation of instruments presupposes that there is a valid
existing contract between the parties, and only the document does not correctly
express the terms of their true agreement.
b. When the real agreement is void, there can be no reformation.
c. ŽŶůLJƚŚĞƐƵďũĞĐƚŽĨƌĞĨŽƌŵĂƚŝŽŶŝƐƚŚĞƌĞĂůŝŶƚĞŶƚŝŽŶŽĨƚŚĞƉĂƌƚŝĞƐ

Practice Questions and Suggested Answers 32


CIVIL LAW REVIEW II

28. Once a contract is perfected, though the same is not in the form required by law, the
contracting parties may compel each other to observe that form.
a. FALSE.
b. If the law requires a document or other special form, the contracting parties may
compel each other to observe that form, once the contract has been perfected. (Art.
1357)

29. Lesion or inadequacy of cause shall not invalidate a contract, unless there has been fraud,
mistake or undue influence.
a. FALSE.
b. džĐĞƉƚŝŶĐĂƐĞƐƐƉĞĐŝĨŝĞĚďLJůĂǁ, lesion or inadequacy of shall not invalidate a contract,
unless there has been fraud, mistake or undue influence. (Art. 1355)

30. The cause in a lucrative contract is the liberality of the benefactor.


a. TRUE.

31. An offer may become ineffective upon insolvency of either party before acceptance is conveyed
even if the insolvency is not judicially declared.
a. TRUE.

32. A contract of sale entered into between spouses is void.


a. FALSE.

MULTIPLE CHOICE QUESTIONS

1. On February 11, 2003, Perfecto offered to sell his house and lot to Reynaldo for P1,500,000.00.
Perfecto told Reynaldo that he was giving Reynaldo up to February 28, 2003 to decide whether or
not to buy the house and lot. Reynaldo accepted the option but did not give anything to Perfecto to
support the option given to him. On May 20, 2003, Perfecto found another buyer who was ready to
buy the house and lot for P2,000,000.00. Perfecto wants to ask you whether he can still withdraw
the offer he made to Reynaldo. Decide.
a. Yes, Perfecto may withdraw the offer because he stands to gain an added profit of
P500,000.00 and this will be more than enough to pay damages to Reynaldo.
b. Yes, Perfecto may withdraw the offer by just informing Reynaldo of such fact.
i. When the offeror has allowed the offeree a certain period to accept, the offer
may be withdrawn at any time before acceptance by communicating such
withdrawal, except when the option is founded upon a consideration.
ii. In this case, the option was not founded upon any consideration.
c. No, Perfecto may not withdraw the offer because the option was accepted by Reynaldo
although Reynaldo did not give anything in support thereof.
d. No, Perfecto may not withdraw his offer until the lapse of the period given to Reynaldo
to exercise his option.

2. Elements that accompany certain contracts unless set aside or suppressed by the parties, are known
as:
a. Natural elements
i. These exist as part of the contract even if the parties do not provide for them
because the law creates them.

Practice Questions and Suggested Answers 33


CIVIL LAW REVIEW II

b. Essential elements
i. These are those that without such, there can be no contract. These are consent,
object and cause.
c. Accidental elements
i. These are those which are agreed upon by the parties and which cannot exist
without being stipulated.
d. Original elements

3. A contract where both parties are required to render reciprocal prestations, are known as:
a. Bilateral contract
b. Unilateral contract
c. Gratuitous contract
d. Commutative contract

4. Contracts take effect only between the contracting parties, their assigns and heirs except in cases
where obligations and rights arising from the contracts are not transmissible by their nature, or by
stipulation or by provision of law. This principle is known as:
a. Relativity of contracts
b. Mutuality of contracts
i. The contract must bind both contracting parties; its validity or compliance
cannot be left to the will of one of them. (Art. 1308)
c. Obligatory force of contracts
i. Obligations arising from contracts have the force of law between the
contracting paties and should be complied with in good faith. (Art. 1159)
d. Liberty of contracts

5. Obligations arising from contracts have the force of law between … (question incomplete)
a.
b.
c. ANSWER
d.

6. One of the following is not a real contract:


a. Pledge
b. Commodatum
c. Lease
i. The contract of lease may be of things or of work and service. (Art. 1642)
d. Mutuum

7. On May 1, 2002, S offered to sell a specific car to B for P500,000.00. B sent his letter of acceptance
to S on May 8, 2002. On May 10, 2002, however, S died in a vehicular accident and his secretary
received the letter or acceptance on May 12, 2002 unaware that S had already died.
a. The contract was perfected on May 8, 2002 when B sent his letter of acceptance.
i. Definitely wrong. Acceptance made by letter or telegram does not bind the
offeror until from the time it came to his knowledge.
b. The contract was perfected on May 12, 2002 when the secretary of S received the
letter.
i. Defnitely wrong. Secretary was not the offeror.

Practice Questions and Suggested Answers 34


CIVIL LAW REVIEW II

c. The contract was not perfected because the offer of S became ineffective when he
died.
i. Under the theory of manifestation, offer and acceptance takes effect only
from the time knowledge is acquired by the person to whom it is directed.
If during the intervening time, the offer or acceptance is extinguished by
death/insanity, such offer or acceptance has no more effect.
d. The contract was perfected on May 1, 2002 because the acceptance made by B on
May 8, 2002 retroacts to the date of the offer.

8. Mistake in three of the following will make a contract voidable. Which one will not?
a. Mistake as to the substance of the thing which is the object of the contract
b. Mistake as to the principal conditions which principally moved one or both parties to
enter into the contract.
c. Mistake as to the identity or qualifications of one of the parties, where identity or
qualifications have been the principal cause of the contract.
d. Simple mistake of account.

In order that mistake may invalidate consent, it should refer to the substance of the thing which is
the object of the contract, or those conditions which have principally moved one or both parties to
enter into the contract.

A simple mistake of account shall give rise to its correction. (Art. 1331)

9. It is the employment of serious or irresistible force to obtain consent.


a. Intimidation
i. There is intimidation when one of the contracting parties is compelled by a
reasonable and well-grounded fear of an imminent and grave evil upon his
person or property, or upon the person or property of his spouses,
descendants or ascendants, to give his consent.
b. Threat
c. Violence
d. Moral coercion

10. It is present when one of the contracting parties is compelled by a reasonable and well-
grounded fear of an imminent and grave evil upon his person or property or upon the person or
property of his spouse, descendants, and ascendants to obtain consent:
a. Violence
b. Physical coercion
c. Intimidation
d. Mistake

Practice Questions and Suggested Answers 35


CIVIL LAW REVIEW II

STATUS OF CONTRACTS

QUESTION ANSWER

1. Contacts undertaken in fraud of creditors when the latter cannot in any Rescissible
other manner collect the claims due them.

2. Sale of private agricultural land to a former Filipino citizen. Valid

3. The contract has a false cause. Void

4. Contract of sale where the consideration is in Chinese Yuan. Valid

5. Oral sale of immovable for P450. Unenforceable

6. Sale of large cattle contained in a private instrument. Void

7. Sale of a guardian of the property of the ward valued at P100,000 for Rescissible
P70,000.
8. Oral lease of a car for 3 years. Valid

9. Oral contract for the sale of a bag where the bag will be delivered after 18 Unenforceable
months and the payment shall be made upon delivery.

10. Absolute deed of sale where the price is not indicated. Valid

11. Contract where the signature of one of the contracting parties is forged. Void

12. One of the contracting parties is relatively incapacitated. Void

13. One of the contracting parties lacks juridical capacity. Void

14. One of the contracting parties is 20 years of age. Valid

15. A contact entered into by a person while insane during a lucid interval. Valid

16. Contract where one party is a minor and the other party is insane. Unenforceable

17. Contacts which refer to things under litigation if they have been entered Rescissible
into by the defendant without the knowledge and approval of the litigants
or of competent judicial authority.

18. … Voidable

19. Contacts where the consent is vitiated by mistake, violence, intimidation, Voidable
undue influence or fraud.

20. A contract entered into in representation of another without authority. Unenforceable

Practice Questions and Suggested Answers 36


CIVIL LAW REVIEW II

SALES

TRUE OR FALSE

1. A thing is determinate when it is particularly designated separating it from all others of the same
class.
a. TRUE.
i. Alternative answer: FALSE, because the statement implies that a thing will
only be determinate if it is particularly designated, excluding things which are
physically segregated.

2. In an absolute sale, ownership of the thing sold shall be transferred to the vendee upon the
delivery thereof.
a. FALSE.
b. The ownership of the thing sold shall be transferred to the vendee upon the actual or
constructive delivery thereof (Art. 1477), OR in any manner signifying an agreement
that possession is transferred from the vendor to the vendee (Art. 1496). (Baviera,
p.63)
c. In sale on approval, ownership passes to the buyer only when he signifies his approval
to the seller or when he does any other act adopting the transaction or when he
retains the goods without giving notice of rejection, or on the expiration of a
reasonable time, if no period has been fixed. (Art. 1502)
d. The parties may stipulate that ownership in the thing shall not pass to the purchaser
until he has fully paid the price. (Art. 1478)
e. Where there is a contract of sale of specific goods, the seller may, by the terms of the
contract, reserve the right of possession or ownership in the goods until certain
conditions have been fulfilled. (Art. 1503)

3. When adverse possession had been commenced before the sale but the prescriptive period is
completed after the transfer, the vendor shall not be liable for eviction.
a. TRUE. Art. 1550.

4. If two or more animals are sold together, whether for a lump sum or for a separate price for
each of them, the redhibitory effect of one shall only give rise to its redhibition, and not that of
the others.
a. FALSE.
b. The statement will only be true if it should NOT appear that the vendee would not
have purchased the sound animal or animals without the defective one. This will be
presumed when a team, yoke, pair, or set is bought, even if a separate price has been
fixed for each one of the animals composing the same. (Art. 1572)

5. For a sale of a thing to be valid, the thing must be determinate.


a. FALSE. The requisite that a thing be determinate is satisfied if at the time the contract
is entered into, the thing is capable of being made determinate without the necessity
of a new or further agreement between the parties. (Art. 1460, par. 2)

6. The price may be in a currency not legal tender here in the Philippines.

Practice Questions and Suggested Answers 37


CIVIL LAW REVIEW II

a. TRUE.

7. A vendor may be liable for eviction even if it is based on a right after the sale.
a. FALSE, the law provides “right PRIOR” to the sale, not after.
i. Alternative answer: TRUE. It is not only based on a right prior to the sale, but
also an act imputable to the vendor.

8. The vendee shall owe interest for the period between the delivery of the thing and the payment
of the price.
a. FALSE. The vendee shall only owe interest in the following instances:
i. Should it have been stipulated;
ii. Should the thing sold and delivered produce fruits or income; or
iii. Should he be in default, from the time of judicial or extrajudicial demand for
the payment of the price. (Art. 1589)

9. Possessory lien and the right of stoppage in transit cannot be exercised by the unpaid seller
simultaneously.
a. FALSE. In cases where there is partial delivery, the seller may exercise his right of
stoppage in transit for those goods which he has already parted with, and his right to
possessory lien on the remainder. (Art. 1528)

10. Goods are considered no longer in transit if the carrier refuses to deliver the goods to the buyer.
a. FALSE. It will only be considered no longer in transit if there was wrongful refusal to
deliver the goods to the buyer or his agent. (Art. 1531[3])

11. The contract is one of barter if the value of the thing to be delivered by one party exceeds the
value of the thing to be delivered by the other party.
a. FALSE. There is barter when it is manifested in the intention of the parties or when the
consideration is partly in money and partly in another thing when the value of the
thing given exceeds the amount of the money or its equivalent. (Art. 1468)

12. A contract of sale is essentially commutative.


a. TRUE. A commutative contract is when there is equivalency in the value of the
prestation to be performed by both parties. Normally, the thing sold would be equal
to the price paid by the other party. (Art. 2010)
b. Gaite v. Fonacier.

13. A sale of a right binds third persons if it is in a public instrument.


a. FALSE. The public instrument must also be registered before it can bind third persons.

14. In a contract to sell, the ownership of the thing automatically passes to the buyer upon the full
payment of the price.
a. FALSE. In a contract to sell, ownership is reserved by the seller despite the delivery to
the buyer. While a contract to sell is considered a special kind of conditional sale, it is
a peculiar kind of sale because despite the happening of the condition and actual
delivery, the buyer does not automatically acquire ownership.
b. If condition happens, the right of the buyer is to compel the seller to execute a final
deed of sale. So ownership does not automatically pass.

Practice Questions and Suggested Answers 38


CIVIL LAW REVIEW II

15. The option money may be considered as part of the price.


a. TRUE. Though it is option money, it can be considered as part of the price as long as it
is stipulated. Without stipulation, the option money cannot be considered as partial
payment because it is a consideration for the option and therefore not part of the
price.

16. A contract of sale is perfected upon the payment of the price.


a. FALSE. A contract of sale is perfected upon the meeting of the minds of the persons as
to the object and the price. (Art. 1475)

17. In a conditional sale, ownership passes to the buyer upon delivery.


a. FALSE. In conditional sales, conditions are imposed by the seller before ownership will
pass. Ownership automatically passes to the buyer from the moment the condition
happens. There is no need for another contract to be entered into.

18. A contract of sale where price is not money is void.


a. FALSE. The cause in a contract of sale may be money or its equivalent.

19. An assignment of credit is a consensual contract.


a. TRUE. An assignment of credit is perfected in accordance with the provisions of Art.
1475, where the law provides that a contract of sale is perfected at the moment there
is meeting of the minds. (Art. 1624)

20. Oral sale of a personal property with a market value of P700 is unenforceable.
a. FALSE. “Market value” is not the same as “price”.

21. When the sale is made through a public instrument, the execution thereof shall be equivalent to
the delivery of the thing which is the object of the contract.
a. FALSE. This will only apply if the contrary intention does not appear on the document.
(Art. 1498)

22. Where goods are shipped and by the bill of lading the goods are delivered to order of the buyer
or of his agent, ownership passes upon the delivery of the goods to the carrier.
a. FALSE. The effect is that seller reserves a right to the possession of the goods as
against the buyer.

23. The goods remain at the seller’s risk until the ownership therein is transferred to the buyer.
a. FALSE. There are instances when ownership has already transferred to the buyer but
the seller will still bear the risk; as when the seller is in delay.

24. Where delivery of the goods has been made to the buyer or to a bailee for the buyer, in
pursuance of the contract and the ownership in the goods has been retained by the seller
merely to secure performance by the buyer of his obligations under the contract, the goods are
at the buyer’s risk from the time of such delivery.
a. TRUE. Art. 1503.

Practice Questions and Suggested Answers 39


CIVIL LAW REVIEW II

25. Where goods are sold by a person who is not the owner thereof, and who does not sell them
under authority or with the consent of the owner, the buyer acquires no title to the goods.
a. FALSE. The buyer may acquire title provided that he bought it in good faith and for
value.

26. Where the seller delivers to the buyer a quantity of goods less than he contracted to sell, and
the buyer accepts or retains the goods as delivered, knowing that the seller is not going to
perform the contract in full, he must pay for them at their fair value.
a. FALSE. If the buyer accepts or retains the goods delivered with knowledge that the
seller is not going to perform the contract in full, he must pay for them at the contract
rate.
b. If however, the buyer has used or disposed of the goods delivered before he knows
that the seller is not going to perform his contract in full, the buyer shall not be liable
for more than the fair value to him of the goods so received. (Art. 1522)

27. Where in pursuance of a contract of sale, the seller is authorized or required to send the goods
to the buyer, delivery of the goods to a carrier, whether named by the buyer or not, for the
purpose of transmission to the buyer is deemed to be a delivery of the goods to the buyer.
a. FALSE. Art. 1523, par. 2.

28. In a lump sum sale of real estate, the vendor shall be obliged to deliver to the vendee, all that
may have been stated in the contract; but should this be not possible, the vendee may choose
between a proportional reduction of the price and the rescission of the contract.
a. FALSE. In the sale of real estate made for a lump sum, there is no increase or decrease
of price allowed although there be greater or lesser areas or number than that stated
in the contract. (Art. 1542)

29. If the same thing should have been sold to different vendees, the ownership shall be transferred
to the person who may have first taken possession thereof in good faith.
a. FALSE. Determine first if the thing sold was movable or immovable.
i. If movable, first who took possession in good faith has the better right.
ii. If immovable:
1. First to register the sale, in good faith; or
2. First to take possession, in good faith; or
3. Party who has oldest title, in good faith.

30. If the possessor of a movable lost or which the owner has been unlawfully deprived, has
acquired it in good faith at a public sale, the owner cannot obtain its return without reimbursing
the price paid therefor.
a. TRUE.

31. The owners of adjoining lands have the right of redemption when a piece of rural land, the area
of which does not exceed one hectare, is alienated.
a. FALSE, because the statement is incomplete; “unless the grantee does not own any
rural land.” (Art. 1621)

Practice Questions and Suggested Answers 40


CIVIL LAW REVIEW II

32. The fruits at the time of redemption will be protected between the redemptioner and the
vendee, giving the vendee the part corresponding to the time he possessed the land in the last
year, counted from the anniversary of the date of sale.
a. FALSE, because the statement is not applicable if there were fruits at the time of sale.
b. For conventional redemption. Can only exist in a sale with a right to repurchase.

33. In the warranty against eviction, the deprivation must be based on a right prior to the sale.
a. FALSE, because the deprivation can also be based on acts imputable to the vendor.

34. The contracting parties may increase, diminish, or suppress the obligation of the vendor to
warrant the object of the sale.
a. TRUE. Warranty is not an essential element of a contract of sale and may therefore be
increased, diminished, or suppressed by the agreement of the parties. (Art. 1548, par.
3)

35. The vendee need not appeal from the decision in order that the vendor may become liable for
eviction.
a. TRUE. Art. 1459.

36. When adverse possession had been commenced before the sale but the prescriptive period is
completed after the transfer, the vendor shall not be liable for eviction.
a. TRUE. Art. 1550.

37. If the property is sold for non-payment of taxes due as a consequence of which the vendee was
evicted, the vendor is liable for eviction.
a. FALSE, because the fact of non-payment of taxes should not be known to the vendee.
Also, the non-payment of taxes cannot be a basis for the vendor’s liability for eviction
because the taxes may become due after the sale. (Art. 1551)

38. Any stipulation exempting the vendor from the obligation to answer for eviction is void, if he
acted in bad faith.
a. TRUE. Art. 1553.

39. The right of pre-emption may be exercised by a co-owner.


a. FALSE, because the right of pre-emption is only exercised by adjoining owner of an
urban land.

40. The right of redemption of co-owners exclude that of adjoining owners.


a. TRUE. Art. 1623.

41. The “contract of sale with a right to repurchase” shall be presumed to be an equitable mortgage
when the vendee remains in possession as lessee or otherwise.
a. FALSE, because it should be the vendor that remains in possession as lessee or
otherwise. (Art. 1602, par. 2)

42. A co-owner of an undivided immovable who sells his share with a right to repurchase to a third
person who subsequently acquires the whole thereof, may be compelled by the latter to
redeem the whole property, if the former wishes to make use of the right of redemption.

Practice Questions and Suggested Answers 41


CIVIL LAW REVIEW II

a. TRUE. Art. 1620.

43. If the vendee made the waiver with knowledge of the risks of eviction and assumed the
consequences, the vendor shall not be liable.
a. TRUE. An equitable mortgage is a transaction that has all the requisites of a mortgage
but is not in the form of a mortgage.

44. The “contract of sale with a right to repurchase” shall be presumed to be an equitable mortgage
when it may be fairly inferred that the real intention of the parties is that transaction shall
secure the payment of a debt or the performance of any other obligation.
a. TRUE. Art. 1602.

45. The “contract of sale with a right to repurchase” shall be presumed to be an equitable mortgage
when the vendor binds himself to pay the capital gains tax.
a. FALSE, because paying capital gains tax, not being a tax on the thing, does not give rise
to the presumption that whoever is paying is the owner. (Art. 1602)
b. The presumption will only arise if the seller bound himself to pay the tax on the thing
not the capital gains tax. That would be the real property tax.

46. By the contract of sale one of the contracting parties transfer ownership over a determinate
thing and the other pays therefore a price certain in money or its equivalent.
a. FALSE, one of the contracting parties is obligated to transfer ownership AND deliver
the thing to the other. (Art. 1475)

47. The vendor must be the owner at the time of delivery for the sale to be valid.
a. FALSE, ownership is not an essential requisite.

48. The ownership of the thing sold shall be transferred to the vendee upon the delivery thereof.
a. FALSE, only applies to absolute sale.

49. The goods will be at the buyer’s risk after the delivery of the goods to the buyer.
a. FALSE, there are exceptions:
i. If there was a stipulation to the contrary (Lawyer’s Cooperative v. Tabora);
ii. There was delay in the delivery.

50. The ownership of the thing shall not pass to the purchaser until he has fully paid the price.
a. FALSE, there are exceptions:
i. Conditional sale
ii. Contract to sell
iii. Executory sale
iv. By stipulation

51. The sale may be valid even if the seller is not the owner of the thing sold.
a. TRUE, as when the sale is by auction or by persons with a statutory power to sell. (Art.
1505, par. 2)

52. A contract of sale may be valid even if the object is neither particularly designated nor physically
segregated from all others of the same class.

Practice Questions and Suggested Answers 42


CIVIL LAW REVIEW II

a. TRUE, the requisite that the thing be determinate is satisfied if at the time the
contract is entered into, the thing is capable of being made determinate without the
necessity of a new or further agreement between the parties. (Art. 1460, par. 2)

53. The price may be in a currency not legal tender here in the Philippines.
a. TRUE.

54. Service may be the subject matter of a valid sale.


a. FALSE, things and rights only.

55. The sale of an immovable for a price less than P500 shall be enforceable even if not in writing.
a. FALSE, sale of an immovable must be in writing regardless of price.

56. In a contract of sale, the seller will have the obligation to take care of the thing sold with the
diligence of a good father of a family.
a. FALSE, this is not absolute. This will not apply if:
i. There was a stipulation on the contrary;
ii. Traditio brevi manu
iii. Sale of a generic thing.

57. When goods are delivered to the buyer on satisfaction, the ownership therein may pass to the
buyer even if the buyer does not signify his approval or acceptance to the seller.
a. TRUE, this may occur when the buyer:
i. Does any other act adopting the transaction;
ii. Retains the goods without giving notice of rejection, then if a time has been
fixed for the return of the goods, on the expiration of time, and if no time has
been fixed, on the expiration of a reasonable time.

58. The risk of loss shall be borne by the owner.


a. FALSE, there are exceptions:
i. If there was a stipulation to the contrary (Lawyer’s Cooperative v. Tabora);
ii. Title was reserved by the seller to secure the payment of the price by the
buyer;
iii. There was delay in the delivery.

59. Dation in payment, whereby property is alienated to the creditor in satisfaction of a debt, shall
be governed by the law of sales.
a. FALSE, dation in payment will only be governed by the law on sales if the debt was in
money.

60. A contract for the delivery at a certain price of an article which the vendor in the ordinary course
of business manufactures or procures for the general market, if the same is on hand, is a
contract for a piece of work.
a. FALSE, the statement pertains to a contract of sale.
b. It is a contract for a piece of work if the seller does not normally produce or
manufacture the thing for the general market. (Art. 1467)

61. A contract of sale with things having a potential existence as object may be a valid sale.

Practice Questions and Suggested Answers 43


CIVIL LAW REVIEW II

a. TRUE. Art 1461.

62. The goods remain at the seller’s risk until the ownership therein is transferred to the buyer, but
when the ownership therein is transferred to the buyer the goods are at the buyer’s risk
whether actual delivery has been made or not.
a. FALSE, there are exceptions:
i. Unless otherwise agreed upon;
ii. Title was reserved by the seller to secure the payment of the price by the
buyer;
iii. If there was delay in the delivery, person who was in delay will bear the loss.

63. In a contract to sell, ownership passes to the buyer upon full payment of the price.
a. FALSE, seller still needs to transfer ownership and deliver.

64. If the possessor of a movable lost or which the owner has been unlawfully deprived has
acquired it in a public sale, the owner cannot obtain its return without reimbursing the price
paid therefor.
a. FALSE, the owner can recover the thing lost without reimbursement if the buyer was
in bad faith. (Art. 559)

65. Where goods are sold by a person who is not the owner thereof, and who does not sell them
under authority or with the consent of the owner, the buyer acquires no better title to the
goods than the seller had.
a. FALSE, this will be true UNLESS the owner of the goods is by his conduct, precluded
from denying the seller’s authority to sell. (Art. 1505)

66. When goods are delivered to the buyer on “sale or return”, ownership is transferred when he
signifies his approval or accepts to the seller or does any other act adopting the transaction.
a. FALSE, this applies to a “sale on approval or satisfaction”.
b. In a “sale or return”, ownership is transferred to the buyer upon delivery but he may
revest the ownership in the seller by returning or tendering the goods within the time
fixed in the contract or within a reasonable time. (Art. 1502)

67. If a movable property is sold to two or more persons, ownership shall belong to the person who
may have first taken possession thereof.
a. FALSE, it must be a possession in good faith. (Art. 1544)

68. Contracts of sale shall be obligatory, in whatever form they may have been entered into,
provided all the essential requisites for their validity are present.
a. FALSE, parties may reciprocally demand performance, subject to the provisions of the
law governing the form of contracts. (Art. 1475)

69. The authority of the agent to sell a piece of land must be in writing; otherwise, the agency is
void.
a. FALSE, sale, not the agency, is void. (Art. 1874)

70. The expenses for registration and execution of the sale are borne by the vendee unless
otherwise stipulated.

Practice Questions and Suggested Answers 44


CIVIL LAW REVIEW II

a. FALSE, expenses are borne by the vendor, not the vendee, unless otherwise
stipulated. (Art. 1487)

71. If the consideration of the contract consists partly in money and partly in another thing, it shall
be considered a barter if the value of the thing given a part of the consideration exceeds the
amount of the money or its equivalent.
a. FALSE, consider the intent of parties first. (Art. 1468)

72. There may be a contract of sale of goods whose acquisition by the seller depends upon
contingency which may or may not happen.
a. TRUE. Art. 1462.

73. The sale of a vain hope or expectancy is void.


a. TRUE. Art. 1461.

74. If the price is simulated, the sale is void, but the act may be shown to have been in reality a
donation or some other act or contract.
a. TRUE. Art. 1471.

75. A contract of sale entered into between a husband and wife is void.
a. FALSE, there are exceptions:
i. If the spouses have a marriage settlement of complete separation of property;
ii. If there is a judicial separation of property. (Art. 1490)

76. A thing is determinate when it is particularly designated.


a. TRUE.

77. For a sale to be valid, the object must be determinate.


a. FALSE. It is sufficient that it can be made determinate at the time the contract is
entered into without the necessity of a new and further agreement between the
parties.

78. A sale is valid even if the consideration to be paid is in Chinese Yuan.


a. TRUE. A price certain may be in MONEY or its equivalent. Money does not have to be
legal tender.

79. Goods are no longer in transit if the carrier or other bailee refuses to deliver the goods to the
buyer or his agent in that behalf.
a. FALSE. It has to be WRONGFUL refusal. (Art 1531)

80. If two or more animals are sold together, whether for a lump sum or for a separate price for
each of them, the redhibitory defect of one shall only give rise to its redhibition, and not that of
the others.
a. FALSE. There may be a redhibition of all animals when it should appear that the
vendee would not have purchased the sound animal or animals without the defective
one. (Art 1572)

Practice Questions and Suggested Answers 45


CIVIL LAW REVIEW II

81. When adverse possession had been commenced before the sale but the prescriptive period is
completed after the transfer, the vendor shall not be liable for eviction.
a. TRUE. Art. 1550.

82. Sellers are liable for breach of warranty.


a. FALSE. Sellers are not always liable for breach of warranty, as when they have
complied with their obligation, and in accordance with their warranties. Further,
warranties may be waived.

83. The vendee shall owe interest for the period between the delivery of the thing and payment of
the price.
a. FALSE. This is only true in the following three cases:
i. should it be stipulated;
ii. should the thing sold and delivered produce fruits and income;
iii. should he be in default, from the time of judicial or extrajudicial demand for
the payment of the price. (Art 1589)

84. Seller may exercise possessory lien and right of stoppage in transitu at the ANY time.
a. FALSE. The right to retain possessory lien presupposes that the seller must still have
possession of the goods. In the right of stoppage in transitu, it is necessary that the
seller has parted with the thing sold and the buyer must be insolvent.

85. Di ko maalala yung isa pa. Something to do with ownership vesting with the buyer at perfection?
a. FALSE.

86. The unpaid seller loses his lien on the goods when he delivers the goods to a carrier or other
bailee for the purpose of transmission to the buyer.
a. FALSE, because there are exceptions to this rule:
i. Seller may reserve ownership to the goods or the right to possession thereof.
(Art. 1529)

87. The seller’s possessory lien or right of stoppage in transitu is not affected by any sale or
disposition of the goods made by the buyer.
a. FALSE, the seller may lose his lien when he assented to the sale or dispositon; or the
goods are covered by a negotiable instrument. (Art. 1535)

88. Notice need not to be given to the original buyer of the intention to resell the goods for the
validity of the resale.
a. TRUE. Art. 1533

89. The right of redemption of co-owners exclude that of adjoining owners.


a. TRUE. Art. 1623

90. If a small piece of rural land which was bought for speculation has been resold, the owner of the
adjoining has a right of redemption at a reasonable price.
a. FALSE, because the statement pertains to rural land when it should be urban land.

Practice Questions and Suggested Answers 46


CIVIL LAW REVIEW II

91. The apparent vendor may ask for the reformation of the instrument in the contracts presumed
to be a sale with a right to repurchase.
a. FALSE, because what should be presumed is an equitable mortgage instead of a sale
with a right to repurchase.

92. If refusal to accept delivery is justified, title to the goods does not pass on to him.
a. TRUE.

93. The remedies under the Maceda Law are alternative.


a. FALSE, remedies under the Maceda Law are cumulative, not alternative. It is the
remedies under the Recto Law that are considered to be alternative.

94. Conventional redemption shall take place when the vendor reserves the right to repurchase the
thing.
a. FALSE, because the word “shall” implies that conventional redemption shall only take
place when the vendor reserves the right to repurchase.

95. In sale with right to repurchase, the vendee is subrogated to the vendor’s rights and actions.
a. TRUE.

MULTIPLE CHOICE QUESTIONS

1. When the goods are delivered to the buyer, the ownership passes to the buyer in:
a. Sale on approval
i. Ownership passes to the buyer when (1) he signifies his acceptance or does
any other act adopting the transaction; (2) retains the goods without giving
notice of rejection, then if a time has been fixed for the return of the goods,
on the expiration of such time, and if no time has been fixed, on the expiration
of a reasonable time. (Art. 1502)
b. Sale on satisfaction
i. Same with sale on approval.
c. Sale on trial
i. Same with sale on approval.
d. Sale or return
i. In a sale or return, the buyer acquires ownership upon delivery of the thing
sold and may revest the ownership to the seller within the time fixed in the
contract or within a reasonable time. (Art. 1502)
e. Contract to sell
i. It is considered a special kind of conditional sale, it is a peculiar kind of sale
because despite the happening of the condition and actual delivery, the buyer
does not automatically acquire ownership. If condition happens, the right of
the buyer is to compel the seller to execute a final deed of sale. So ownership
does not automatically pass.

2. It is an affirmation of fact or any promise by the seller relating to the thing which has a natural
tendency to induce the buyer to purchase the same, relying on such promise or affirmation.
a. Condition
b. False representation

Practice Questions and Suggested Answers 47


CIVIL LAW REVIEW II

c. Warranty
i. Art. 1546
d. Seller’s talk

3. S makes an offer to B on January 1, 1996. B makes known his acceptance in a letter sent on
January 2, and received by the secretary of S on January 10. Meantime, on January 5, S died.
a. There is no contract.
i. An offer becomes ineffective upon the death of either party before acceptance
is conveyed. (Art. 1323)
ii. The offer is deemed accepted once the seller has knowledge of the buyer’s
acceptance.
b. The contract is voidable because a party is insane.
c. There is already a meeting of minds, the contract is perfected.
d. The contract is unenforceable.

4. G was appointed guardian of S, the latter being 16 years old, S sold his parcel of land in writing
to B valued at P100,000 for P65,000. What is the status of the contract?
a. Rescissible
b. Void
c. Unenforceable
d. Enforceable
e. Voidable
i. One of the parties who entered into the contract was incapable of giving
consent.

5. Which of the following obligations of the vendor cannot be waived?


a. To allow the buyer to examine the goods sold
b. To pay the expenses of the deed of sale
c. To warrant the thing sold
d. To transfer ownership to the buyer

6. When delivery takes place by the mere consent of agreement of contracting parties as when the
vendor merely points to the thing sold which shall thereafter be a the control and disposal of
the vendee if the thing sold cannot be transferred to the possession of the vendee at the time of
the sale, delivery is effected:
a. By actual delivery
i. Delivery or transfer of a thing from hand to hand if it is movable, or by certain
material and possessory acts of the grantee performed in the presence and
with the consent of the grantor if it is movable.
ii. “real tradition”
b. By traditio longa manu
i. The grantor pointing out to the grantee the thing which is delivered which at
the time must be within sight.
c. By traditio brevi manu
i. Delivery of movable property takes place when the vendee had the thing
already in his possession before the sale took place, not as owner but as
lessee, borrower or as depositary.
d. Constitutum possesorium

Practice Questions and Suggested Answers 48


CIVIL LAW REVIEW II

i. Vendor remains in possession of the property sold by virtue of a lease contract


agreement with the vendee.

7. Lucy is a lessee of a store in a shopping mall. She only sells/assigns her rights to Salome. She
then allows Salome to occupy the stall. The delivery here is called:
a. Quasi-tradition
i. This is used to indicate the exercise of a right by the grantee with the
acquiescence of the grantor.
b. Actual delivery
i. Delivery or transfer of a thing from hand to hand if it is movable, or by certain
material and possessory acts of the grantee performed in the presence and
with the consent of the grantor if it is movable.
ii. “real tradition”
c. Traditio longa manu
i. The grantor pointing out to the grantee the thing which is delivered which at
the time must be within sight.
d. Traditio brevi manu
i. Grantee is already in possession of the thing under a title which is not of
ownership, such as when the lessee purchases from the lessor the object of
the lease.

8. Ownership of the thing sold is transferred/acquired/retained:


a. Retained by the seller in “sale or return”
i. Wrong. In a sale or return, ownership passes to the buyer upon delivery, but
the buyer can revest the title to the seller.
b. Transferred to the buyer upon constructive or actual delivery of the thing sold
c. Acquired by the buyer upon perfection of the contract
i. Wrong. It is fundamentally wrong. There was no delivery, but only mere
perfection of the contract. As a general rule, ownership is transferred upon
delivery.
d. Transferred to the buyer upon acceptance of the price
i. Wrong. It is fundamentally wrong. There was no delivery, but only mere
perfection of the contract. As a general rule, ownership is transferred upon
delivery.

9. Which of the following statements is correct:


a. In a contract of sale, full payment of the price is in the nature of suspensive condition in
that the seller is obligated to transfer ownership of the thing sold upon payment.
i. Wrong. This only applies in an absolute sale. In a conditional sale, full payment
of the price may not be the only condition to be fulfilled.
b. The seller need not be the owner of the thing sold at the perfection of the sale.
i. This is not absolutely correct. There are different types of perfection which
requires immediate delivery.
ii. What the law requires of the seller is the right to transfer ownership, not
necessarily ownership of the thing sold.
c. There may be a transfer of ownership over the thing even if the seller has not
delivered the thing sold to the buyer.

Practice Questions and Suggested Answers 49


CIVIL LAW REVIEW II

d. In a contract of sale, the buyer becomes the owner of the thing sold upon full payment
of the purchase price.
i. Wrong because this does not cover conditional sales.

10. Which of the following statements is not correct:


a. Actual delivery of the thing or payment of the price is not required for the perfection of
the sale.
i. The contract of sale is perfected at the moment there is a meeting of the
minds upon the thing which is the object of the contract and upon the price.
(Art. 1475)
b. A stipulation that even when the object is delivered to the buyer, ownership will not
pass until the price is fully paid is not valid.
c. A sale is consummated upon delivery of the thing and the payment of the purchase
price.
i. In a conditional sale, payment of the price and/or delivery of the thing sold
may not be the only conditions to be fulfilled for the consummation of the
sale.
d. Sales through letters or telegrams are deemed perfected only when acceptance by the
buyer is known to the seller.

11. The delivery of movable property may be made by the mere consent or agreement of the
contracting parties, if the thing sold cannot be transferred to the possession of the vendee at
the time of the sale. This mode of delivery is known as:
a. Actual delivery
b. Traditio constitutum possesorium
i. Vendor remains in possession of the property sold by virtue of a lease contract
agreement with the vendee.
c. Traditio longa manu
d. Traditio brevi manu
i. Grantee is already in possession of the thing under a title which is not of
ownership, such as when the lessee purchases from the lessor the object of
the lease.
e. Formal delivery

12. In a sale of an incorporeal property, the following are the modes of delivery, except:
a. Execution of public instrument
b. Placing of the titles of ownership in the possession of the vendee
c. Use by the vendee of his rights, with the vendor’s consent
d. Actual delivery
e. None of the above

13. In this kind of sale, the ownership passes to the buyer on delivery, but he may revest the
ownership in the seller by returning or tendering the goods within the time fixed in the contract,
or, if no time has been fixed, within a reasonable time.
a. Conditional sale
i. Conditions are imposed by the seller before ownership will pass. Normally, the
condition is the full payment of the price.

Practice Questions and Suggested Answers 50


CIVIL LAW REVIEW II

ii. Ownership automatically passes to the buyer from the moment the condition
happens. There is no need for another contract to be entered into.
b. Sale or return
c. Sale on trial
i. Ownership passes to the buyer when (1) he signifies his acceptance or does
any other act adopting the transaction; (2) retains the goods without giving
notice of rejection, then if a time has been fixed for the return of the goods,
on the expiration of such time, and if no time has been fixed, on the expiration
of a reasonable time. (Art. 1502)
d. Contract to sell
i. It is considered a special kind of conditional sale, it is a peculiar kind of sale
because despite the happening of the condition and actual delivery, the buyer
does not automatically acquire ownership. If condition happens, the right of
the buyer is to compel the seller to execute a final deed of sale. So ownership
does not automatically pass.
e. Absolute sale
i. Seller does not reserve his title over the thing sold and thus, upon delivery of
the thing, ownership passes regardless of whether or not the buyer has paid.

14. For ownership to pass, the seller must have the right to sell:
a. Before the perfection of the sale
b. At the time of perfection
c. Upon payment
d. At the time ownership is to pass
e. Upon registration of the sale

15. When a person who is not the owner of a thing sells or alienates and delivers it, and later the
seller or grantor acquires title thereto, such title passes by operation of law to the buyer or
grantee. This is:
a. Estoppel in pais
i. By the principle of estoppel, a person is precluded from denying that another
person has authority to sell because of his acts. Also known as “Estoppel in
Pais” which is a kind of equitable estoppel because of the acts /
representation of the owner, he may not later on deny the authority of the
3rd person.
b. Estoppel by deed
i. When the seller who was not the owner at the time of the sale, acquires
ownership, automatically, ownership passes to the buyer by operation of law.
However, Article 1434 requires delivery to the buyer.
c. Estoppel by record
i. Case where the seller was estopped from denying the authority to sell due to
his prior testimony made in court.
d. Equitable estoppel
e. None of the above

16. In a CIF arrangement, delivery is considered to take place at:


a. Place of perfection of the sale
b. Place of payment

Practice Questions and Suggested Answers 51


CIVIL LAW REVIEW II

c. Port of origin
d. Port of destination
i. This applies in an FOB arrangement.
e. None of the above
17. In this mode of delivery, the seller would remain in possession of the thing after the sale:
a. Actual delivery
b. Traditio constitutum possesorium
c. Traditio longa manu
d. Traditio brevi manu
e. Formal delivery

18. This seller has a statutory power to sell:


a. Sheriff
i. This seller has the authority given by the court.
b. Agent
i. This seller was given the authority to sell by the owner.
c. Auctioneer
d. Notary public
i. He may be the owner but he may have the authority of the law to sell, known
as “Statutory Power to Sell” (Article 1505). Examples: Notary public in pledge,
liquidators, guardians and receivers.

19. Ownership passes upon delivery in:


a. Conditional sale
i. Conditions are imposed by the seller before ownership will pass.
b. Sale on trial
i. Ownership passes to the buyer when (1) he signifies his acceptance or does
any other act adopting the transaction; (2) retains the goods without giving
notice of rejection, then if a time has been fixed for the return of the goods,
on the expiration of such time, and if no time has been fixed, on the expiration
of a reasonable time. (Art. 1502)
c. Absolute sale
i. Seller does not reserve his title over the thing sold and thus, upon delivery of
the thing, ownership passes regardless of whether or not the buyer has paid.
d. None of the above

20. Who can transfer ownership by virtue of a sale?


a. Seller
b. Buyer
i. Consider “sale or return”
c. One with a right to sell
d. All of the above
e. None of the above

21. The following are requisites for a valid exercise of the right of stoppage in transitu, except:
a. seller is unpaid
b. goods are in transit
c. seller has not parted with the possession of the goods

Practice Questions and Suggested Answers 52


CIVIL LAW REVIEW II

d. buyer is insolvent

22. One of the following is not a remedy granted to an unpaid seller:


a. right of stoppage of goods in transit
b. right of lien over the goods
c. right of resale
d. right to demand a security for the payment of the price

Under the Civil Code, the following are the remedies of an unpaid seller: (1) retain
the thing in his possession; (2) stoppage in transitu; (3) resale; and (4) rescission.
There is no mention of a right to demand security for the payment of the price.

23. One of the following is not an alternative remedy available to a seller of personal property sold
on instalment if the buyer defaults.
a. to go to court and ask the court to order delivery of the property
b. to foreclose the chattel mortgage
c. to sue for specific performance
d. to take possession of the property and forfeit the amount paid by buyer

24. The following are the alternative remedies, except one, available to the buyer in breach of
warranty by the seller.
a. keep the goods and ask for damages
b. refuse to accept the goods and ask for damages
c. rescind the sale and retain the goods

The remedy provided by law is to rescind the sale and refuse to receive the
goods, or if the goods have already been received, return them or offer to
return them to the seller and recover the price or any part thereof which has
been paid. (Art. 1599)

d. keep the goods and set up against the seller by way of recoupment in price

25. If redemption is made, which of the following will not be paid by the seller to the buyer?
a. price paid by the buyer
b. expenses in the execution of the sales contract paid by the buyer
c. all necessary expenses on the thing sold and to be redeemed
d. interest of the price paid by the buyer

The law does not require the redemptioner to pay for any interest.

26. M, N and O are co-owners of a parcel of land proindiviso. M sold his 1/3 share in the land to N in
an absolute sale. Which is correct?
a. The deed of sale between M and N is void because it was not made in favor of a third
party.
b. O may exercise his right of redemption on the interest sold by M to N.
c. O may redeem only ½ of the interest sold by M to N.
d. O cannot exercise the right of redemption because the sale was made in favor of a co-
owner.

Practice Questions and Suggested Answers 53


CIVIL LAW REVIEW II

27. An unpaid seller has the following rights, except:


a. a lien on the goods after he has parted with the possession of them

Unpaid seller cannot have a lien on the goods if he does not have possession
over them.

b. in case of insolvency of the buyer, a right of stopping the goods in transitu


c. a right to rescind, under the circumstances provided by law
d. a right of resale, under the circumstances provided by law

28. The unpaid seller is not entitled to retain the possession of the goods where:
a. the goods have been sold without stipulation as to credit
b. the goods have been sold on credit, but the term of credit has expired
c. the buyer is insolvent
d. the possession of the seller on the goods is only as agent or bailee for the buyer
e. none of the above

Art. 1527.

29. What mode of extinguishing a contract of sale is effected when a person is subrogated upon the
same terms and conditions stipulated in the contract in the place of one who acquires a thing by
onerous title?
a. compensation
b. conventional redemption
c. novation
d. legal redemption
e. none of the above

30. A sold to B a specific car for P 250,000 in 10 equal monthly installments. B failed to pay the 3 rd
installment. The right of A is:
a. cancel the sale
b. demand payment from B
c. (missing)
d. (missing)

31. In a contract of sale, a warranty against eviction is:


a. an essential element
b. a natural element
c. an accidental element
d. a formal requisite

32. It is an affirmation of fact or promise by the seller relating to the thing which has a natural
tendency to induce the buyer to purchase the same, relying on such promise or affirmation.
a. condition
b. false representation
c. express warranty
d. seller’s talk

Practice Questions and Suggested Answers 54


CIVIL LAW REVIEW II

33. In a contract of sale of a specific carabao, the seller was obliged to deliver the animal on
December 31, 2003. In November 2003, the carabao had an offspring. The offspring belongs to:
a. Seller, because the sale is already perfected prior to its truth.
b. Seller, because the fruit arose before the obligation to deliver arises.
c. Buyer, because the fruit arose after the perfection of the contract.
d. Buyer, if he pays the purchase price.

Under Art. 1164, the creditor has a right to the fruits of the thing from the
time the obligation to deliver it arises. However, he shall acquire no real right
over it until the same has been delivered to him.

34. There is no warrant in:


a. “as is, where is sale”
a. Sale which means as it is found, where it is found
b. sale of second hand articles
c. animals sold at fairs
d. sale by virtue of authority in fact or law
e. none of the above

35. Persons not liable for breach of warranty, except:


a. Sheriff
b. Auctioneer
c. Mortgagee
d. Pledgor
e. none of the above

36. When does the buyer of a thing has the right to the fruits of the thing bought? From the time
the obligation to deliver the thing arises.

37. If an immovable property is sold to two persons, ownership shall belong to the person ____.
Who first recorded the sale and bought the property in god faith.

38. A contract of sale is perfected ____.

As a general rule, when there is meeting of the minds between the vendor and the
vendee on the object of the sale and the price certain.

39. It is a contract by virtue of the terms which the parties thereto promise and obligate themselves
to enter into another contract at a future time, upon the happening of certain events, or the
fulfillment of certain conditions.

Contract of option or option-contract.

40. A contract of sale is not necessarily:


a. Consensual
b. Bilateral
c. Commutative

Practice Questions and Suggested Answers 55


CIVIL LAW REVIEW II

d. Onerous
e. None

41. When the goods are delivered to the buyer, the ownership passes to the buyer in ___.

Sale or return.

42. The following may not be valid objects of a contract of sale, except:
a. objects outside the commerce of men
b. illicit things
c. Future goods.
d. impossible service

43. …

44. S makes an offer to B on January 1, 1996. B makes known his known his acceptance in a letter
sent on January and received by S on January 10. Meantime on January 5, S became insane.

There was no perfected contract of sale since there was no meeting of the minds
between the party, applying the cognition theory where there is a perfected contract
of sale if the acceptance of the buyer is made known to the seller. Unlike
manifestation theory where acceptance by the buyer perfects the contract already.

45. Which of the following obligations of the vendor cannot be waived.

to deliver the thing to the buyer

Transfer of ownership of the object of the sale is necessary in the performance of the
obligation of the vendor and transfer of ownership is effected only through delivery.
There is always delivery in a contract of sale it is either actual or constructive.

46. Ownership of the thing sold is transferred/acquired/retained:


a. retained by seller in sale or return
b. transferred to buyer upon constructive or actual delivery
c. acquired by the buyer upon perfection of contract
d. transferred to buyer upon acceptance of price

47. Which of the following statements is correct?


a. in COS, the full payment of the price is in the nature of suspensive
b. seller need not be the owner of the thing sold at the perfection of the sale
c. there may be a transfer of ownership over the thing even if the seller has not delivered
the thing sold to the buyer
d. in COS, the buyer becomes the owner of the thing sold upon full payment of the
purchase price

48. In a contract of sale of a specific carabao, the seller was obliged to deliver the animal on Dec 31
2003. In November 2003, the carabao had an offspring. The offspring belongs to ___.

Practice Questions and Suggested Answers 56


CIVIL LAW REVIEW II

Seller, because the fruit arouse before the obligation to deliver arises.

49. Sale is distinguished from dation in payment in that in dation in payment ___.

There is a pre-existing obligation or credit.

50. There is a contract of sale ___.


a. When goods are delivered to distributor on consignment
b. When the manufacturer delivers the foods to an agent where the former retains the
ownership and dictates the term of the sale
c. When foods are delivered to a person on charge account
d. When goods are delivered to an agent to be sold by him and the agent is not liable to
the manufacturer of the goods

51. The following persons cannot acquire by purchase, even at a public or judicial auction, either in
person or through the mediation of another, except ___

Agents, the property whose administration or sale may have been entrusted to them.

52. Araneta wrote a letter to Bascon wherein he offered to sell a piece of land. In Araneta’s letter he
gives Bascon a period of 2 months within which to pay the price of 500k. After 50 days, Araneta
told Bascon that he is increasing the price of the land to 700k. Can Bascon compel Araneta to
accept the 500k first offered by Araneta and execute the deed of sale?

No, Bascon did not signify his acceptance of offer.

53. A characteristic of COS which involves exchange of value, it is ___.

Commutative

54. A case where COS may not necessarily be in writing to be enforceable.


a. Sale of 100 piculs of sugar at 400 per picul when there is partial delivery; even if there
is no partial delivery, the amount is already more than 500 and covered by the statue
of frauds.
b. Sale whereby its term cannot be performed within one year from the making thereof
c. Sale of land
d. All of the Above
e. None of the Above

55. In a contract of sale, the nature of the object of the sale will not be relevant in
a. Maceda Law
b. Recto Law
c. Double Sales
d. Statute of Frauds
e. None of the Above

Practice Questions and Suggested Answers 57


CIVIL LAW REVIEW II

STATUS OF CONTRACTS

1. The guardian purchases the property of his ward.


a. VOID.

2. The purchaser of a private agricultural land is a former Filipino citizen.


a. VALID.

3. Contracts of sale with a false cause.


a. VOID.

4. Sale of a car where the price is in Chinese Yuan.


a. VALID.

5. Oral sale of an immovable for P450.


a. UNENFORCEABLE.

6. Sale of large cattle in a private instrument.


a. VOID.

7. The guardian sold the property of his ward valued at P100,000 to another person for the price of
P70,000.
a. UNENFORCEABLE.

8. A deed of absolute sale which does not contain a provision as to the price.
a. VALID.

9. Sale of hereditary rights of an heir to the executor of the will of the decedent.
a. VOID.

10. Sale of a property in litigation to the court stenographer of the court where the case is pending.
a. VOID.

Practice Questions and Suggested Answers 58


CIVIL LAW REVIEW II

SALES II

MODIFIED TRUE OR FALSE

A. Both are true


B. Both are false
C. No. 1 is true, No. 2 is false
D. No. 1 is false, No. 2 is true

1. Statements:
· No affirmation of the value of the thing, nor any statement purporting to be a statement of the
seller’s opinion only, shall be construed as a warranty.
o FALSE, it will be considered a warranty when the seller made such affirmation or
statement as an expert and it was relied upon by the buyer. Art. 1546.

· The sale of animals suffering from contagious diseases shall be void.


o TRUE, Art. 1575.

2. Statements:
· For the vendor to be held liable for breach of warranty against eviction, the judgment must be
based on a right prior to the sale.
o FALSE, judgment may also be based on an act imputable to the vendor.

· The vendee need not appeal from the decision in order that the vendor may become liable for
eviction
o TRUE, Art. 1549.

3. Statements:
· When adverse possession had been commenced before the sale but the prescriptive period is
completed after the transfer, the vendor shall not be liable for eviction.
o TRUE, Art. 1550.

· There are no implied warranties in sale of animals at fairs or at public auctions, or of live stock
sold as condemned.
o FALSE, only warranties against hidden defects are not provided for such sales.

4. Statements:
· If the property is sold for nonpayment of taxes due, the vendor is liable for eviction.
o FALSE, such nonpayment of taxes due shall not be made known to the vendee before
the sale.

· If two or more animals are sold together, whether for a lump sump or for a separate price for
each of them, the redhibitory defect of one shall only give rise to its redhibition and not that of
the others.
o FALSE, the statement will only be true if it should NOT appear that the vendee would
not have purchased the sound animal or animals without the defective one. This will

Practice Questions and Suggested Answers 59


CIVIL LAW REVIEW II

be presumed when a team, yoke, pair, or set is bought, even if a separate price has
been fixed for each one of the animals composing the same. Art. 1572.

5. Statements:
· Any stipulation exempting the vendor from the obligation to answer for eviction shall be void.
o FALSE, vendor must act in bad faith to render such stipulation void. Art. 1553.

· Should the vendee have made the waiver with knowledge of the risks of eviction and assumed
its consequences, the vendor shall not be liable.
o TRUE, Art. 1554.

Practice Questions and Suggested Answers 60


CIVIL LAW REVIEW II

MULTIPLE CHOICE QUESTIONS

6. The following cannot be held liable for breach of warranty except:


a. sheriff
b. auctioneer
c. mortgagor ??
d. pledgee
e. none of the above

The law would specifically exempt certain persons from liability for breach of warranty like
sheriff, auctioneer, mortgagee, pledgee and other persons who sell by virtues of an authority
of law like notary public because they are not really selling for themselves, they are selling on
behalf of another person.

7. The redhibitory action, based on the faults or defects of animals must brought within:
a. forty days from the date of their delivery to the vendee. (Art. 1577)
b. forty days from the date of the contract
c. six months from the date of their delivery to the vendee.
d. six months from the date of the contract
e. one year from the date of the contract

8. If there is a stipulation exempting the vendor from the obligation to answer for eviction and the
vendor acted in bad faith the vendor shall be liable for the following:
a. Value of the thing at the same time of eviction
b. Income or fruits, if he has been ordered to deliver them to the party who won the suit against
him
c. Expenses of the contract, if the vendee has paid them, and
d. Damages and interest, and ornamental expenses, if the sale was made in bad faith
e. All of the above. (Art. 1555)

9. There is no implied warranty as to hidden defects/quality in the following, except:


a. “as is, where is” sales
b. sale of second hand articles
c. sale by virtue of authority in fact or law (Art. 1570)
d. animals sold at fairs or at public auctions (Art. 1574)
e. when validly suppressed by parties

10. The buyer is deemed to have accepted the goods


a. when he intimates to the seller that he is accepting them.
b. when he does any act in relation to the goods which is inconsistent with the ownership of the
seller
c. when he retains the goods after the lapse of a reasonable time without intimating to the
seller that he has rejected them.
d. all of the above. (Art. 1585)
e. none of the above.

Practice Questions and Suggested Answers 61


CIVIL LAW REVIEW II

MACEDA LAW

TRUE OR FALSE

1. If the place of payment should not have been stipulated, the payment must be made at the
place of the delivery of the thing sold.
2. The buyer of goods is not bound to accept delivery by installments.
3. Where the goods are delivered to the buyer, he is not deemed to have accepted them unless
and until he has had a reasonable opportunity of examining them for the purpose of
ascertaining whether they are in conformity with the contract.
4. Where there is a breach of warranty by the seller, the buyer may, refuse to accept the goods,
and maintain an action against the seller for damages for the breach of warranty.
5. Acceptance of the goods by the buyer shall not discharge the seller from liability in damages or
other legal remedy for breach of any promise or warranty in the contract of sale.
6. If, after acceptance of the goods, the buyer fails to give notice to the seller of the breach in any
promise of warranty within a reasonable time after the buyer knows, or ought to know of such
breach, the seller shall not be liable therefor.
7. The Maceda Law can only be invoked if the buyer had paid at least two years of installments
before default.
8. The right to a grace period shall be exercised by the buyer, under the Maceda Law, only once in
every five years of the life of the contract, and its extensions, if any.
9. Down payments, deposits or options in a sale covered by the Maceda Law, shall be included in
the computation of the total number of installment payments made.
10. A sale of realty on credit will be covered by the Maceda Law.

MULTIPLE CHOICE QUESTIONS

11. The buyer is deemed to have accepted the goods when:


a. he intimates to the seller that he has accepted them
b. when the goods have been delivered to him, and he does any act in relation to them which is
inconsistent with the ownership of the seller.
c. when, after the lapse of a reasonable time, he retains the goods without intimating to the
seller that he has rejected them.
d. all of the above
e. none of the above

12. The vendee shall owe interest for the period between the delivery of the thing and the payment
of the price, in the following cases, except:
a. Should it have been so stipulated
b. Should the thing sold and delivered produce fruits or income
c. From the time the obligation to pay the price becomes due and demandable
d. None of the above

13. The Realty Installment Buyer Act may apply to:


a. a sale of industrial lots
b. a sale of commercial buildings
c. sales to tenants
d. all of the above

Practice Questions and Suggested Answers 62


CIVIL LAW REVIEW II

e. none of the above

14. Under the Maceda Law, the buyer is entitled to a grace period of not less than:
a. 30 days
b. 60 days
c. 90 days.
d. one year
e. two years

15. The cash surrender value is a minimum of:


a. 20% of the total payments made
b. 30% of the total payments made
c. 40% of the total payments made
d. 50% of the total payments made
e. 92% of the total payments made

Practice Questions and Suggested Answers 63


CIVIL LAW REVIEW II

STATUS OF CONTRACTS OF SALE

1. There may be a contract of sale of goods, whose acquisition by the seller depends upon a
contingency which may or may not happen.
a. TRUE. Express provision 1462(2).

2. The sale of expectancy is voidable.


a. FALSE. The sale of vain hope or expectancy is void. (1461(2))

3. Things having a potential existence may be the object of the contract of sale.
a. TRUE. Express provision 1461(1).

4. A contract of sale is void when the object is neither particularly designated nor physically
segregated from all others of the same class.
a. FALSE. The requisite that a thing be determinate is satisfied if at the time the contract
is entered into, the thing is capable of being made determinate without the necessity
of a new or further agreement between the parties. (1460(2))

5. If the price is simulated, the sale is void, but the act may be shown to have been in reality a
donation, or some other act or contract.
a. TRUE. Express provision 1471.
i. Alternative answer: FALSE. Parties may be held bound by the false price
indicated in the instrument, especially when the interest of the Government
or third persons would be adversely affected by the reformation of the
contract. (Spouses Doromal v. CA)

6. The husband and the wife cannot sell property to each other.
a. FALSE. The following are exceptions to the rule – a
i. Separation of property in the marriage settlement; and
ii. Judicial separation of property (Art. 1490)

7. A contract of sale entered into by a guardian over the property of his ward is rescissible if the
price is only 70% of the value of the property.
a. TRUE.

8. A contract of sale is void when the executor is the vendee and the object of the sale is a
property of the estate under his administration.
a. TRUE. Relative Incapacity. (Art. 1491, par. 3)

9. A contract of sale if void when the vendee is not a Filipino citizen and the object of the sale is a
private land.
a. FALSE. An exception is when a formerly natural-born Filipino citizen who lost his
Filipino citizenship. (Estate of Serra v. Heirs of Hernaez)
i. Alternative answer: TRUE. The question depends on how you appreciate “not
a Filipino citizen”.

Practice Questions and Suggested Answers 64


CIVIL LAW REVIEW II

10. If the fixing of the price is left to the discretion of one of the parties, the contract will not be
perfected.
a. FALSE. The fixing of the price can never be left to the discretion of one of the
contracting parties. However, if the price fixed by one of the parties is accepted by the
other, the sale is perfected. (1473)

Practice Questions and Suggested Answers 65


CIVIL LAW REVIEW II

REMEDIES

TRUE OR FALSE

16. It is not essential to the validity of a resale that notice of the time and place of such resale
should be given by the seller to the original buyer.
a. TRUE.

17. It is not essential to the validity of resale that notice of an intention to resell the goods be given
by the seller to the original buyer.
a. TRUE.

18. The unpaid seller cannot directly or indirectly buy the goods in the exercise of the right to resell.
a. TRUE.

19. The seller of goods is deemed to be an unpaid seller when a bill of exchange or other negotiable
instrument has been received as conditional payment.
a. FALSE.

20. The unpaid seller may exercise his right of stoppage in transitu by obtaining actual possession of
the goods.
a. TRUE.

21. The unpaid seller shall not be liable to the original buyer for any profit made by the resale of the
goods.
a. TRUE.

22. A lien on the goods of the unpaid seller of goods can be exercised even if he is in possession of
them merely as a depository.
a. TRUE.

23. The remedies of the unpaid seller of goods can no longer be invoked if ownership in the goods
have passed already to the buyer.
a. FALSE.

24. The seller of goods is deemed to be an unpaid seller when the whole of the price has not been
paid.
a. TRUE.

25. The remedies of an unpaid seller are alternative in character.


a. FALSE.

MULTIPLE CHOICE QUESTIONS

26. That the buyer is insolvent is a requirement for the exercise of the:

Practice Questions and Suggested Answers 66


CIVIL LAW REVIEW II

a. A right to retain the goods


b. A right of stopping the goods in transitu
c. A right of resale
d. A right to rescind the sale
e. A right to demand for the payment of the unpaid price

27. The unpaid seller of goods loses his lien , except:


a. When he delivers the goods to a carrier or other bailee for the purpose of transmission
to the buyer without reserving the ownership in the goods or the right to the possession
thereof
b. When the buyer or his agent lawfully obtains possession of the goods.
c. When he has obtained judgment or decree for the price of the goods.
d. By waiver thereof
e. None of the Above

28. Goods are no longer in transit:


a. If the buyer, or his agent in that behalf, obtains delivery of the goods before their arrival
at the appointed destination
b. If, after the arrival of the goods at the appointed destination the carrier or other bailee
acknowledges to the buyer or his agent that he holds the goods on his behalf and
continues in possession of them as bailee for the buyer or his agent
c. If the carrier or other bailee wrongfully refuses to deliver the goods to the buyer or his
agent in that behalf
d. All of the above
e. None of the Above

29. An unpaid seller cannot exercise the right to resell if:


a. The seller has parted possession of the goods
b. The goods are of perishable nature
c. The seller expressly reserves the right of resale in case the buyer should make default
d. The buyer has been in default in the payment of the price for an unreasonable time
e. All of the above
f. None of the above

30. The unpaid seller’s right of lien or stoppage in transitu is not affected by any sale, or other
disposition of the goods which the buyer may have made, unless:
a. The seller has parted possession of the goods
b. A negotiable document of title has been issued for goods, which were sold to a
purchaser for value in good faith to whom such document has been negotiated
c. The goods are of perishable nature
d. The seller expressly reserves the right of resale in case the buyer should make default
e. The buyer has been in default in the payment of the price for an unreasonable fare

Practice Questions and Suggested Answers 67


CIVIL LAW REVIEW II

EXTINGUISHMENT OF CONTRACT OF SALE

TRUE OR FALSE

1. Legal redemption shall take place when the vendor reserves the right to repurchase the thing
sold.
a. FALSE.

2. The vendor who repurchases the thing shall respect the leases constituted by the vendee.
a. FALSE.

3. Legal redemption is the right to be subrogated, upon the same terms and conditions stipulated
in the contract, in the place of one who acquires a thing by purchase only.
a. FALSE.

4. Should there have been no fruits at the time of the sale and some exist at the time of
redemption, they shall be prorated between the redemptioner and the vendee.
a. TRUE.

5. The contract shall be presumed to be a sale with a right to repurchase when the price in a
mortgage is unusually inadequate.
a. FALSE.

6. The vendor cannot avail himself of the right of repurchase without returning to the vendee the
price of the sale and the necessary and useful expenses made on the thing sold.
a. TRUE.

7. In case of doubt, a contract purporting to be a sale with right to repurchase shall be construed
as an equitable mortgage.
a. TRUE.

8. The vendee is subrogated to the vendor’s rights and actions.


a. TRUE.

9. The creditors of the vendor cannot make use of the right of redemption against the vendee,
until after they have exhausted the property of the vendor.
a. TRUE.

10. The right of redemption of co-owners excludes that of adjoining owners.


a. TRUE.

MULTIPLE CHOICE QUESTIONS

11. If the transaction is an equitable mortgage, the remedy is:


a. declaration of nullity

Practice Questions and Suggested Answers 68


CIVIL LAW REVIEW II

b. annulment
c. rescission
d. any of the above
e. none of the above

12. In conventional redemption, the period for redemption, in the absence of an express agreement
shall last:
a. one year from the date of the contract
b. four years from the date of the contract
c. one year from the registration of the contract
d. four years from the registration of the contract
e. none of the above

13. Should there be an agreement as to the period of redemption, the period cannot exceed:
a. one year
b. four years
c. ten years
d. thirty years
e. none of the above

14. In a sale of a parcel of land with a right to repurchase, there being no stipulation to the contrary,
ownership passes to the vendee upon:
a. the perfection of the sale
b. upon delivery
c. upon payment of the price
d. upon the lapse of the period to repurchase
e. upon the order of the court

15. The remedy of the creditors of the vendor as to the right of redemption against the vendee is:
a. a cumulative remedy
b. a principal remedy
c. a subsidiary remedy
d. all of the above
e. none of the above

Practice Questions and Suggested Answers 69


. A and B entered into a contract of sale of A’s lot. B delivered an earnest money evidenced by a
receipt which stated that the same shall be forfeited if the buyer failed to pay on or before January
31, 2005 and the seller can sell it to another. Possession remained with A. What is the nature of
the contract? Explain.

ANSWER:

A contract of sale is a consensual contract. A contractual contract is perfected by mere consent.


Hence, by the mere agreement of the parties over the object of the contract and over the price or
consideration, the contract of sale is deemed to be perfected. Such contract of sale may be a
conditional obligation because of the imposition of conditions for the efficacy of the obligation to
pay or deliver the object of sale of the consideration.

In the given case, although earnest money had already been delivered by B, yet the
possession of the lot still remained with A, the contract of sale still remains a valid contract of
sale. The actual delivery of the subject matter or payment of the price agreed upon are not
necessary components to establish the existence of a valid contract of sale; and their
non-performance do not also invalidate or render void a sale taht has began to exist as a valid
contract at perfection.

2. In a Contract of Sale, there was a failure to pay the consideration. Upon request of the buyer,
the seller granted an extention to pay, but the buyer failed to pay within the extended period,
hence, the seller availed of the rights of rescission. Is rescission proper? Why?

ANSWER:

It depends. If it is a sale of an immovable property, under Art. 1592 of the Civil Code,
even though it may have been stipulated that upon failure to pay the price at the time agreed
upon the rescission of the contract shall of right take place, the vendee may pay, even after the
expiration of the period, as long as no demand for rescission of contract has been made upon the
vendee judicially or by a notarial act. As such, demand for rescission by suit or notarial act must
first be made by the vendor. With respect to movable property, Art. 1593 provides that the vendor
can rescind the contract, as a matter of right, if the vendee, without any valid cause, does not (1)
accept delivery; or (2) pay the price.

OBLIGATIONS AND CONTRACTS

128. Distinguish between civil and natural obligations.

Civil and natural obligations may be distinguished as follows: (1) civil obligations derive their
binding force from positive law; while natural obligations derive their binding effect from equity and natural
justice; and (b) civil obligations can be enforced by court action or the coercive power of public authority,
the fulfillment of natural obligations cannot be compelled by court action but depends exclusively upon the
good conscience of the debtor.

129. Give the concept of a ​quasi-contract​. Distinguish it from the other sources of
obligations.

A ​quasi-contract is a juridical relation which arises from certain lawful, voluntary, and unilateral
acts, to the end that no one may be unjustly enriched or benefited at the expense of another.
The act giving rise to a ​quasi-contract must be lawful, thereby distinguishing it from crime in which
the act or omission is unlawful; it must be voluntary, thus differentiating it from a ​quasi-delict​, which is
based on fault or negligence or mere lack of foresight; and it must be unilateral, to distinguish it from
contract in which there are two parties who come to an agreement.

130. A telephone company and an electric cooperative entered into a contract subject
to the following terms and conditions: (a) the telephone company is allowed to use the electric
posts of the electric cooperative for the installation of its telephone and cable wires; (b) the
telephone company is obliged to install and allow the electric cooperative the free use of ten (10)
telephone lines; and (c) the contract: “​shall be for as long as the telephone company has need for
the electric posts and shall terminate only if the electric cooperative is forced to stop or abandon
its operations as a public service and it becomes necessary to remove the electric posts​.”

Is the contract between the telephone company and the electric cooperative subject to a
potestative condition?

No. The condition that the contract shall be effective for as long as the telephone company has
need for the electric posts is decidedly a potestative condition because it is dependent upon the sole will
of the telephone company. But the condition that such contract may be terminated if the electric
cooperative is forced to stop or abandon its operations is a casual condition which depends on chance,
hazard, or the will of a third persons. In sum, the contract between the telephone company and the
electric cooperative is subject to a mixed condition, that is, a condition dependent partly on the will of a
party and partly on chance, hazard, or the will of a third person, which do not invalidate a contract ​(Naga
Telephone Company, Inc. vs. Court of Appeals, 230 SCRA 351).

131. In December 1985, X and Y executed a Deed of Conditional Sale wherein the former
agreed to sell his 500-square meter lot in Baguio City to the latter for P1 Million payable P100,000
as down payment , and the balance 60 days after the squatters on the property have been
removed. If the squatters are not removed within six months, the P100,000 down payment shall be
returned by the vendor to the vendee.

X filed an ejectment suit against the squatters, but in spite of the decision in his favor, the
squatters would not leave. In August 1986, X offered to return the P100,000 down payment to Y, on
the ground that he is unable to remove the squatters on the property. Y refused to accept the
money and demanded that X execute a deed of absolute sale of the property in his favor, at which
time it will pay the balance of the purchase price. Incidentally, the value of the land had doubled
by the time.

X consigned the P100,000 in court, and filed an action for rescission of the deed of
conditional sale, plus damages.

Will the action prosper?

No, the action will not prosper. The action for rescission may be brought only by the aggrieved
party to the contract. Since it was X who failed to comply with his conditional obligation, he is not the
aggrieved party who may file the action for rescission but Y. Y, however, is not opting to rescind the
contract but has chosen to waive X’s compliance with the condition which he can do under Article 1545 of
the Civil Code.

132. On May 10, 1983, X, as vendor, and Y, as vendee, executed an “Agreement of


Purchase and Sale” respecting two parcels of agricultural land for the total sum of P2 Million. The
agreement contains the following terms and conditions. (1) Y shall pay the amount of P600,000.00
as down payment; (2) the balance of P1.4 Million shall be paid in four (4) equal quarterly
installments of P350,000.00, the first to be due and payable on June 15, 1983, and every quarter
thereafter, until the whole amount is paid; (3) that upon full payment of the purchase price, X shall
execute a good and sufficient deed of sale and conveyance in Y’s favor; and (4) that immediately
upon the signing of the agreement, Y shall take possession of the parcels of land.

On May 15, 1983, Y took possession of the subject parcels of land together with all the
improvements thereon. And to answer for his balance of P1.4 Million, Y issued four post-dated
checks payable to X in the amount of P350,000.00 each dated June 15, 1983, September 15, 1983,
December 15, 1983 and March 15, 1984. When presented for payment, however, the checks were
dishonored due to insufficient funds. Y promised to replace the checks but failed to do so.
Aggrieved by Y’s failure and refusal to make good the checks, X filed suit to rescind the
agreement and to recover the properties. X contented that the failure of Y to completely pay the
purchase price is a substantial breach of the obligation which entitles him to rescind under Article
1191 of the Civil Code. Y countered by citing Article 1383 of the Civil Code to the effect that where
specific performance is available as a remedy, rescission may not be resorted to.

Will the action prosper?

Yes, the action will prosper. Rescission of reciprocal obligations under Article 1191 of the Civil
Code should be distinguished from rescission of contract under Article 1383. although both presupposes
and both require mutual restitution when proper, they are not entirely identical.

While Article 1191 uses the term “rescission,” the proper term really is “resolution.” Resolution is
a principal action which is based on the breach of a party of his obligation, while rescission under Article
1383 is a subsidiary action limited to cases of rescission for ​lesion under Article 1381 of the Civil Code.
Obviously, the contract entered into by the parties in the case at bar does not fall under any of those
mentioned by Article 1381. consequently, Article 1383 is inapplicable.

A reading of the agreement entered into by X and Y shows that it is in the nature of a contract to
sell, as distinguished from a contract of sale. In a contract of sale, the title to the property passes to the
vendee upon the delivery of the thing sold; while in a contract to sell, ownership is, by agreement,
reserved in the vendor and is not to pass to the vendee until full payment of the purchase price. In a
contract to sell, the payment of the purchase price is a positive suspensive condition, the failure of which
is no a breach, casual or serious, but a situation that prevents the obligation of the vendor to convey title
from acquiring an obligatory force.

X in the case at bar bound himself to deliver a deed of absolute sale and clean title covering the
two parcels of land upon full payment by Y of the purchase price of P2 Million. This promise to sell was
subject to the fulfillment of the suspensive condition that Y will pay in the full the purchase price. Y,
however, failed to complete payment of the purchase price. The non-fulfillment of the condition of full
payment rendered the contract to sell ineffective without force and effect. Failure of Y to pay, in this
instance, is not even a breach but merely an event which prevents X’s obligation to convey title from
acquiring binding force. Hence, the agreement of the parties may be set aside, but not because of a
breach on the part of Y for failure to complete payment of the purchase price. Rather, his failure to do so
brought about a situation which prevented the obligation of X to convey title from acquiring obligatory
force ​(Ong vs. Court of Appeals, 310 SCRA 1).

133. X sold a parcel of land to Y. Their notarized contract of sale contains a provision
that the seller warrants that he will defend the buyer’s title to and ​peaceful possession of the
property. There were existing lessees in the subject property at the time of the sale. Buyer now
wants to cancel the contract for violation of a condition. He alleges that he cannot use the lot
because of the seller’s failure to eject the lessees. He also claims that the provision in the contract
of sale where the seller warranted to defend his title to and peaceful possession of the property
partakes of the nature of a condition.
(a) Is the buyer’s claim tenable?

The buyer’s claim is not tenable. The buyer failed to appreciate the difference between a
condition and a warranty and the consequences of such distinction. The failure of the seller to eject the
lessees from the lot in question and to deliver actual and physical possession thereof to the buyer cannot
be considered a substantial breach for two reasons: First, such “failure” was not stipulated in the contract
as a condition – whether resolutory or suspensive; and second, its effects and consequences were not
specified either. The stipulation adverted to by the buyer does not impose a condition or an obligation on
the part of the seller to eject the lessees from the lot. Said stipulation pertains merely to the usual
warranty against eviction, and not a condition that was not met. Had the parties intended to impose on the
seller the obligation to eject the tenants from the lot sold, the same should have been expressly provided
for in the contract. For example, the contract may provide that if the lessees are not ejected with a certain
period, the contract can be rescinded ​(Power Commercial and Industrial Corp. vs. Court of Appeals, 274
SCRA 597).

(b) Was there a breach of warranty against eviction?

There was no breach of warranty against eviction. The buyer was not deprived of his title. The
presence of lessees does not even constitute an encumbrance on the land nor does it deprive the buyer
of its control thereof.

134. X leased to Y a building for a period of fifteen years, at a monthly rental of


P10,000.00 payable in advance during the first ten days of each month. The lessee occupied the
premises and made payments of rentals for about three (3) years, after which no rentals were
paid. X, through his lawyer, required Y to return the leased property if he cannot pay the rents.
Pursuant to this demand, Y vacated the property and notified X, and the latter took possession
thereof. X later brought an action for the recovery of the rentals that should be paid under the
lease contract for the unexpired term of twelve (12) years.

Will the action prosper?

The action will not prosper. Under Article 1191 of the Civil Code, X was permitted to elect
between the two remedies of resolution or specific performance, with damages in either case. He is not
entitled to pursue both of these inconsistent remedies. In this case, X clearly elected to resolve or rescind
the lease contract.

In the common case of the resolution or rescission of a contract of sale for failure of the
purchaser to pay the stipulated price, the seller is entitled to be restored to the possession of the thing
sold, if it has already been delivered. But he cannot have both the thing sold and the price which was
agreed to be paid, for the resolution or rescission of the contract has the effect of destroying the obligation
to pay the price. Similarly, in the case of the resolution of a contract of lease, the lessor is entitled to be
restored to the possession of the leased premises, but he cannot have both the possession of the leased
premises for the remainder of the term and the rent which the other party contracted to pay. The
termination of the lease has the effect of destroying the obligation to pay for the future.

135. On October 25, 1980, C loaned to D the sum of 100,000.00 with interest at the rate
of 25% per annum for the term of two (2) years. To guarantee the loan, D pledged some pieces of
jewelry. On December 5, 1991, D brought an action to recover the jewels, upon payment of the
amount of the loan and interest. C contends that the action to recover the jewels had already
prescribed because the period of prescription must be computed from the date of the contract of
loan (1980) inasmuch as from that date D could already have recovered the jewels by paying the
amount of the loan with interest.
Has the action prescribed?

The action has not yet prescribed. There being no circumstance to show that the period in the
contract of loan was for the benefit of either party, it must be presumed to be for the benefit of both. And it
must be so, for this is a case of loan, with interest, wherein the term benefits D by the use of the money
lent and C by the interest. This being so, D had no right to pay the loan before the lapse of two (2) years,
without the consent of C, because such a payment in advance would have deprived the latter of the
benefit of the stipulated interest. It follows that the action to recover the thing pledged accrued only from
the date of maturity of the loan, or on October 25, 1982. Consequently, the 10-year period of prescription
on written contract, such as the one involved in the problem, has not yet expired when the action was
brought.

136. What is an alternative obligation?

An alternative obligation is one where out of two or more prestations which are due, the
performance of one is already sufficient compliance with the obligation ​(Art. 1199, par. 1).

137. X borrowed money from Y. It was agreed that at the maturity of the debt, X will give
Y either the sum lent or a particular house and lot. Is the stipulation valid?

Yes, the stipulation is valid because it is simply an alternative obligation, which is expressly
allowed by the law. The agreement to convey the house and lot in the event of X’s failure to pay the debt
in money at its maturity does not constitute ​pactum commissorium​. It is not an attempt to permit the
creditor Y to declare a forfeiture of the security upon the failure of the debtor to pay the debt at maturity. It
is simply provided that if the debt is not paid in money it shall be paid in another specific way by the
transfer of the property at a valuation.

138. What is a facultative obligation?

A facultative obligation is one where only one prestation has been agreed upon but the obligor
may render another in substitution ​(Art. 1206, par. 1).

139. Distinguish between alternative and facultative obligations.

Alternative obligation and facultative obligations may be distinguished as follows:

1. In an alternative obligation, various things are due, but the giving of one is already
sufficient compliance with the obligation. In a facultative obligation, only one thing is principally due, and it
is that one which is generally given, but the substitute may be given to render payment or fulfillment easy;

2. If one of the presentations in an alternative obligation is illegal and the other prestations
are valid, the obligation remains. In a facultative obligation, if the principal obligation is void, there is no
longer any need of giving the substitute.

3. In an alternative obligation, if it is impossible to give all except one, that last one must still
be given. In a facultative obligation, if it is impossible to give the principal, the substitute does not have to
be given, and if it is impossible to give the substitute, the principal must still be given;

140. A and B signed a promissory note binding themselves to pay C, jointly and
severally, the amount of P30,000.00. For non-payment of the debt, C sued A and B for sum of
money. After trial, judgment was rendered in C’s favor directing A and B to pay the indebtedness.
The judgment did not state whether the liability of the defendants was joint or solidary. C then
asked for execution on the properties of A for the whole obligation. In the contract, liability was
solidary but in the judgment, nothing was said about the nature of the obligation.
How should the obligation be considered, joint or solidary?

The obligation should be considered as merely a joint one, hence, C can get the properties of A
corresponding to his proportionate share in the judgment debt. The judgment did not state that the
obligation was joint and several, so none of the defendants may be required to pay for the whole
obligation.

141. X, as lessor, and Y, as lessee, entered into a contract of lease. Their lease
agreement contains the following terms: (a) the leased premises shall be used exclusively by the
lessee for a specific purpose; (b) the lessee is strictly prohibited from using the leased premises
for any other purpose without the written consent of the lessor; (c) the lessee shall not sublease
his right of lease over the leased premises; (d) the lessor shall have the right to sell the leased
premises during the period of the lease; and (e) in the event that the lessor decides to sell the
leased premises, the lessee has the first option to purchase the property.

On the second year of the lease, the lessor informed the lessee of his desire to sell the
property. Being then sickly, the lessee assigned his right of first option under the lease contract
to his daughter.

Does the daughter have the right to exercise her father’s right of first option under the
lease contract?

The assignment has no legal basis. Article 1311of the Civil Code is too clear to be misinterpreted.
It provides that “contract take effect only between the parties, their assigns and heirs except in cases
where the rights and obligations arising from the contract are not transmissible by their nature or by
stipulation or by provision of law.”

In the case at bar, the lease contract between the lessor and the lessee clearly reveals the intent
of the parties to limit their lease relationship to themselves alone. It cannot be denied that the lessee’s
right of first option to buy the leased property in case of its sale is but part of the bigger right to lease said
property from the lessor. The option was given to the lessee because he was the lessee of the property. It
was a component of the consideration of the lease. If the lessee is barred by the contract from assigning
his right to lease the property to any other party, he is similarly barred from assigning his first option to
buy the leased property to his daughter ​(Bayangan vs. Court of Appeals, 278 SCRA 379)​.

142. May a solidary creditor assign his right under the obligation?

Under Article 1213 of the Civil Code, a solidary creditor cannot assign his right without the
consent of the others. This is so because a solidary obligation implies mutual agency and mutual
confidence. Should the assignee or substitute do acts which would prejudice the others, there is no doubt
that the right of the other creditors would be endangered, hence, the necessity of their consent.

143. H died intestate in 1977 survived by his widow, W, and his three legitimate
children, X, Y and Z. In the settlement of the intestate estate of H, W and her children executed a
notarized Deed of Partition by virtue of which each of them received a share in the intestate estate
of H consisting of several parcels of registered lands. In their partition agreement, W and her
children were assigned individual parcels of land, at two parcels each, as their respective shares
in the inheritance. A day after the execution of the partition agreement, however, W and her
children executed a Memorandum of Agreement by virtue of which they agreed to divide equally
among them the proceeds of the sale of the lots that were assigned to each of them.

In 1981, a decision was rendered against W in an action for sum of money filed by her
creditors. When the decision became final, a writ of execution was issued and the sheriff levied
upon the two (2) lots assigned to W. When W refused to surrender the owner’s duplicate copies of
the titles of the lots, the creditors petitioned the court to nullify said titles. X, Y and Z opposed the
petition on the ground that they are co-owners of the lots in question by virtue of their
Memorandum of Agreement which they claimed novated their partition agreement.

(a) Was there a novation of the Deed of Partition?

The contention of X, Y and Z that the Memorandum of Agreement novated the Deed of Partition
is not well-taken. The Memorandum of Agreement fell short of producing a novation because it does not
express a clear intent to dissolve the old obligation as a consideration for the emergence of a new one.
Likewise, X, Y, and Z failed to show that the Deed of Partition and the Memorandum of Agreement are
materially and substantially incompatible with each other. Indeed, the Deed of Partition granted title to the
lots in question to W to whom they were assigned, and the Memorandum of Agreement created an
obligation on the part of W to share with the other co-owners the proceeds of the sale of such lots. There
is no incompatibility between these two contracts; hence, no novation.

(NOTE: ​Novation, one of the modes of extinguishing an obligation, requires the concurrence of
the following: (1) there is a previous valid obligation; (2) the parties concerned agree to a new contract;
(3) the old contract is extinguished; and (4) there is a valid new contract. Novation may be express or
implied. In order that an obligation may be extinguished by another which substitutes the same, it is
imperative that it be so declared an unequivocal terms (express), or that the old and the new obligations
be on every point incompatible with each other (implied​).

(b) Was the co-ownership revived by the execution of the Memorandum of


Agreement?

The very provisions of the Memorandum of Agreement belie the revival of the co-ownership. First,
it retains the partition of the properties which W and her children supposedly placed in the co-ownership;
and second, it vests in the registered owner the power to dispose of the lots adjudicated to him or her
under the Deed of Partition even without the consent of the other parties to the Memorandum of
Agreement. ​Jus disponendi is an attribute of ownership, and only the owner can dispose of a property
(Cruz vs. Court of Appeals, 293 SCRA 239).

144. X delivered to Y pieces of jewelry to be sold on commission basis. Despite the


lapse of the period agreed upon, Y failed to return the jewelry items to X. When sued for estafa, Y
interposed the defense that her agreement with X was novated when the latter agreed to be paid
directly by the buyers of the jewelry items and on installment basis. She added that her liability is
merely civil in nature.

Was there a novation of X and Y’s agreement?

The acceptance by X of partial payments tendered by Y’s buyers does not evince the intention of
X to have their agreement novated. It was simply necessitated by the fact that, at that time, Y had
substantial accounts payable to X. Thus, to obviate the situation where X would end up with nothing, she
was forced to receive the tender of the buyers. It is thus easy to see why X’s acceptance of the buyer’s
payment on installment basis cannot be construed as a case of either ​expromision or ​delegacion
sufficient to justify the attendance of extinctive novation. Not too uncommon is when a stranger to a
contract agrees to assume an obligation; and while this may have the effect of adding to the number of
persons liable, it does not necessarily imply the extinguishment of the liability of the first debtor. Neither
would the fact alone that the creditor (such as X) receives guaranty or accepts payments from a third
person who has agreed to assume the obligation, constitute an extinctive novation, absent an agreement
that the first debtor shall be released from responsibility.
Granting that there was such novation, the criminal liability for estafa committed by Y is then not
affected by the subsequent novation of the contract, for it is a public offense which must be prosecuted
and punished by the State ​(Quinto vs. People, 305 SCRA 708).

145. X, as lessor, and ABC Bank, as lessee, entered into a contract of lease involving a
building. Their lease agreement specifically states that the “term of the lease shall be fourteen (14)
years commencing from April 1, 1978 and may be renewed for a like term at the option of the
lessee.” X contends that this stipulation is void for being violative of the principle of mutuality of
contract under Article 1308 of the Civil Code.

(a) Is the clause “may be renewed for a like term at the option of the lessee” violative
of the principle of mutuality of contracts?

The fact that the lessee’s option to renew the lease is binding only on the lessor and can be
exercised only by the lessee does not render such option void for lack of mutuality. After all, the lessor is
free to give or not to give the option to the lessee. And while the lessee has a right to elect whether to
continue with the lease or not, once he exercises his option to continue and the lessor accepts, both
parties are thereafter bound by the new lease agreement. Their rights and obligations become mutually
fixed, and the lessee is entitled to retain possession of the property for the duration of the new lease, and
the lessor may hold him liable for the rent therefore. Mutuality obtains in such a contract and equality
exists between the lessor and the lessee since they remain with the same faculties in respect to
fulfillment.

(b) How shall the clause be interpreted or applied?

The clause “​may be renewed for a like term at the option of the lessee​” means that the exercise
by the lessee of his option resulted in the automatic extension of the contract of the lease under the same
terms and conditions prevailing in the original contract of lease; i.e., for fourteen (14) years, the phrase
“​for a like term​” referring to the period of the lease. If the renewed contract were still subject to mutual
agreement by the lessor and the lessee, then the option – which is an integral part of the consideration for
the contract – would be rendered worthless. For then, the lessor could easily defeat the lessee’s right by
simply imposing unreasonable and onerous conditions to prevent the parties from reaching an agreement
(Allied Banking Corp. vs. Court of Appeals, 284 SCRA 357).

1. The essence of a contract of sale is


a. ​ownership
b. possession
c. warranty
d. delivery of object

2. If there is a meeting of the minds, there is a perfected contract of sale.


a. Yes, because all consensual contracts are perfected by mere consent
b. No, because delivery of the thing is essential for the perfection of the contract of sale
c. Yes, because the obligation of the seller to sell and buyer to buy arise as soon as there is meeting of
the minds.
d. No, because meeting of the minds is not the only requisite of the perfection of a contract of
sale

3. a buyer may compel the seller to make the delivery if


a. when what is to be delivered is an indeterminate thing
b. when what is to be delivered is a generic thing
c.​ when what is to be delivered is a determinate thing
d. when what is to be delivered is a specific thing

4. in a contract of sale of personal property, if the price is payable in installments, the vendor may
exercise the following remedies
a. ​if the vendee fails to pay the full amount, vendor can demand exact fulfillment of the
obligation
b. if the vendee fails to pay an installment, vendor can cancel the sale
c. if the vendee fails to pay 2 or more installments, he may demand rescission and keep amounts paid
so far
d. both a and b

5. Can a person without capacity to act can enter into a contract of sale?
a. No, because for a contract of sale to be valid, both parties should have the capacity to act.
b. Yes, because anyone is free to buy anything especially in department stores
c. No, because it would constitute lack of consent making the contract voidable
d.​Yes, because there are some things that are indispensable for the buyer's clothing or dwelling

6. in a contract of sale between husband and wife, who can contest its validity?
a. either the husband or the wife
b. persons claiming to be illegitimate children of either the husband or wife
c. ​the siblings of either husband or wife (reason: HEIRS of husband/wife)
d. the state when there are creditors of either husband or wife

7. the following persons cannot acquire by purchase, even at a public or judicial auction, either in person
or through the mediation of another except
a. the guardian in connection with the property of the person under their guardianship.
b. t​he lawyer in connection with the property of his client not in litigation
c. a public officer in connection with the property of the state under his administration
d. agents in connection with the property interested to him for sale

8. all things which are not outside the commerce of men may be the object of a contract of sale except for
a. future things
b. ​future inheritance
c. rights which are not intransmissible
d. an undivided interest

9. when payment is made in a contract of sale


a. the contract of sale is consummated already
b. ​title passes over to the vendor
c. possession (either actual or constructive) is deemed of the vendee's
d. the contract of sale is perfected already

10. Can a person who is not an owner of a thing alienate and deliver it?
a. No, when he doesn't have the authority to do so.
b. ​Yes, when he eventually acquires title of the thing
c. No, when the thing is acquired in bad faith by the vendor
d. Yes, when he is eventually in possession of the thing

11. A sold books to B as evidenced by invoice receipts who in turn sold the books to C. However, B
turned out to be an impostor and never paid A. Who is the rightful owner of the books?
a. A because title never passed to B thus C also didn't acquire the title
b. B because title already passed to him which he couldn't pass to C for B's non-payment to A.
c. ​C because title already passed to him by operation of law. EDCA publishing v. Santos (B
had a voidable title. actual delivery to impostor because there was no unlawful deprivation. the
books were disposed of with the owner's consent even though there was fraud)
d. C because possession was with him already.

12. A sold B a car with the understanding that B would pay after the car is registered in his name. After
the registration and the deed of sale were delivered to B, B disappeared with the car which was used by A
to deliver the documents. B in turn sold the car to C who bought it in good faith. Who had better right of
the car?
a. ​A because the title never passed to B C also didn't acquire the title. (Aznar v. Yapdiangco: B
had no title of car. there was no delivery of car thus title is non-existing thus void. There was no
transfer of ownership as A was dispossessed of the car without his consent.)
b. B because title already passed to him which he couldn't pass to C for B's non-payment to A.
c. C because title already passed to him by operation of law.
d. C because possession was with him already.

13. When can the courts fix the price in a contract of sale?
a. When parties are both unable or unwilling to fix the price
b. ​When third persons acted in bad faith or by mistake. (1469)
c. When one of the parties acted in bad faith or by mistake.
d. When third persons are prevented from fixing the price by fault of the seller or the buyer.

14. A sold his land originally valued at P10,000 to B for P1 and some unspecified services stated in the
contract as consideration. Is the sale valid?
a. Yes, because aside from the one-peso consideration, there were other services to be rendered.
b​. No, because there is no real price and no valid consideration. (Bagnas v CA. no specified
services. 1 peso here is a notoriously low indicator of actual value)
c. Yes, because the price is real and agreed upon by both parties thus there was no fraud involved.
d. No, because there was false consideration.

15. The fixing of the price can be left to the discretion of one of the contracting parties if
a. it's the seller who fixed the price because he's the owner after all
b. it's the buyer who fixed the price but the seller later consented to the price.
c. it's the seller who fixed the price but the buyer later consented to the price.
d. it's the either the seller or buyer who fixed the price and the other consented to it. (1473)

16. A sold B his van. After agreeing on the price, both A and B failed to agree on how the payment should
be made. But, B made a downpayment nevertheless which A accepted. When B demanded the delivery
of the van, A refused claiming that there was no perfected contract of sale . B insisted that he made a
downpayment already. Was there a perfected contract of sale?
a. Yes, because there was a downpayment made already thus the obligation to consummate the contract
arose already.
b. No, because there was no perfected contract of sale as both parties failed to agree on how the
payment should be made. (Velasco v CA. Manner of payment must be agreed upon.)
c. Yes, because the downpayment was accepted by A already.
d. No, because there was no perfected contract of sale as there was no consent by A.

17. A told B he wanted to buy his land thus A paid a reservation money which B accepted evidenced by
the receipt with the indication that A would buy it within the year. When the period expired B sold the land
to C. A demanded that B should deliver the land to him already and tried paying the amount which B
refused. Was B obligated to sell the land to A after the expiration of the period?
a. Yes, because money had already been paid which was accepted by B even though the period to buy it
expired already
b. No, because what A had was only a privilege to buy which period already expired. (Limson v.
CA. this is only an option money and not an earnest money. there was nothing in the receipt that
would indicate that the amount was part of purchase price. period expired already)
c. Yes, because there was money paid already which was accept by B and there was the eventual
payment of the full price which B refused.
d. No, because A as the owner had the right to sell the property before the payment of the land.

18. A told B he wanted to sell his land to him. B said he would buy the land as soon as it can be
registered. After registration, B refused to buy the land already. Was there a perfected contract of sale?
a. Yes, because there was an offer which was accepted by B already thus A can demand for
specific performance. (De la Cavada v Diaz: promise was accepted right away and the promise
was mutual. a promise made by one party, if in accordance with the form required by the law may
be a good consideration for a promise made by another)
b. No, because it was only an option to buy with a condition.
c. Yes, because the condition was fulfilled already thus A can demand for specific performance.
d. No, because there was no meeting of the minds as to the sale of the land but only to the condition of
the eventual sale of the land.

19. A was a lessee of B's building with an agreement of right of first refusal should the building be sold.
However, B sold the building to C. What is A's remedy?
a. File a suit for specific performance because there was an existing agreement of right of first refusal.
b. File an action for rescission of the contract of B and C on ground of fraud and his right of first refusal
c. File an action for recovery of damages against B for violation of the right of first refusal (Ang Yu
v CA - can't file for SP. It's only a right.)
d. File an action for recognition of the right of first refusal and ask the court for rescind the contract of B
and C.

20. Delivery transfer title of the property in the following cases except:
a. ​when the seller isn't the owner of the property
b. when payment is made in installments.
c. when there's no full payment of price yet
d. when on a sale on trial, he fails to give the goods back upon expiration of period

21. A bought logs from B with the agreement that B would deliver the logs alongside the vessel. While in
transit, the vessel sank thus A sued B for recovery of payment made for the logs. Will the suit prosper?
a. Yes, because B had the responsibility of keeping the logs in good condition until it was delivered to A.
b. No, because there was no negligence on the part of A. (---although this could also be an answer------)
c. Yes, because there was no delivery yet to A. But B will have an action for recovery against the vessel
owner.
d. ​No, because B complied with his responsibility already, by delivery the logs in good condition
alongside the vessel. (Bean v Cadwaller)

22. The following are warranties of the seller of the document of title except:
a. that the document is genuine
b. that he as knowledge of no fact which would impair the validity or worth of the document
c. that he has a right to transfer the title to the goods
d. that the property sold is not fake nor an imitation

23. The following are remedies of an unpaid seller except:


a. possessory lien where the seller reserves his right to possession or ownership of property regardless of
delivery
b. when goods are in transit and the negotiable instrument has been issued, to have the goods
redelivered to him (1532 par. 2)
c. right of resale where goods are perishable in nature or where seller expressly reserves that right
d. right to rescission of the contract of sale

24. A bought machines for production of Christmas decorations from B which he promised to deliver by
October. On the first week of October, A signed the contract of sale and believed the promises of B that
the machines were on their way already. However, when it still didn't arrive on the first week of
November, A made demands. The machines never arrived that year. What remedy could A have against
B?
a. an action for specific performance as there was delay already and B didn't comply with it yet.
b. ​action for rescission of the contract because despite demands, there was still no delivery (soler
v cheeky: time is of the essence here (machines in time for Christmas season, he signed the
contract believing that the machines were on their way)
c. action for recovery of damages because the right of A vested by B to have it on time (for the Christmas
season) was violated by B.
d. action for specific performance and damages because there was delay and the right to have it on time
was violated.

25. The vendee loses his right to make use of a term in the following except:
a. when he becomes insolvent
b. when he fails to pay two or more installments
c. when there are no guaranties
d. when he violates his undertaking in consideration of period

26. A bought a condominium unit from B worth 10,500,000 measuring 90 square meters in area. After
payment had been made, possession and ownership was turned over to A. When A transferred, he found
out that the unit was only 75 square meters big. Although he still continued to occupy the property, A
demanded refund from B for the difference of 15 square meters (1,750,000). Is B obligated to give the
excess?
a. Yes, because clearly there was fraud. The contract explicitly stated 90 square meters
b. No, because there was negligence on the part of A. He should have inspected and surveyed the
property even before full payment
c. Yes, because there was an error or mistake in fact as to the measurement.
d. No, clearly, the condo unit was purchased for a lump sum and not for a piece rate as stated in
the contract that the condo unit was for 10,500,000. (1542 and 1539)

27. A bought a residential house from B for 50 million pesos payable in 10 years at at a bi-annual
installment payment scheme. On the sixth year, A failed to pay an installment but was only able to pay it
the following year. However, on the eigth year, A again failed to pay. This time, after repeated demands
from B, A failed to pay. B filed an action for rescission of contract. Will the suit prosper?
a. No, because he had already paid for more than 7 years. He had paid more than half already.
b. Yes, because he failed to pay the 2nd installment despite repeated demands but he is entitled for a
refund for the payments made.
c. No, because according to the Maceda Law, a one month grace period can be given for every year paid.
d. ​Yes, because he has already been in delay twice in five years, although he is entitled to a
refund. (Maceda law, grace period can only be used once in every five years)

28. A sold an antique jar to B explaining to him that acquired it from C, a certified expert antique dealer
and relaying to him what C said that it was already 200 years old and made from real jade. Relying on A's
explanations, B bought it. However, when B had it inspected by an antique dealer, the jar turned out to
be fake. A filed an action for recovery of payment on ground of fraud of the contract. Will the suit
prosper?
a. No, because there was no fraud to begin with as B agreed to purchase it relying solely on A
who wasn't an antique dealer. (1340, 1341 - B wasn't the antique dealer but C. A relied only on B,
not C)
b. Yes, because B relied on A's explanation which came from C, an expert on antique jars
c. No, because B should have it checked before buying it
d. Yes, because an expression of opinion doesn't signify fraud unless made by an expert an C clearly was
an expert in antique jars.

29. when the warranty against eviction has been agreed upon or nothing has been stipulated on this
point, in case eviction occurs, the vendee shall have the right to demand of the vendor the ff. except:
a. the return of the value which the thing sold had at the time of the eviction be it greater or less than the
price of the sale
b. the damages and interests, and ornamental expenses if the sale was made in bad faith
c. the costs of from breach of contract of sale (1551: correct one is the cost of suit which caused
the eviction)
d. the income or fruits if has been ordered to deliver them to the party who won the suit against him.

30. A bought a motorcycle from B with a factory machine as the mortgage. Payment was to be made in
installments. However, A failed to pay 3 installments already thus B chose to foreclose on the machine.
When the machine still didn't satisfy the entire payment of the motorcycle, B wanted to foreclose on the
2nd mortgage which was made after the 1st default on the installment. A said he couldn't foreclose on the
2nd mortgage anymore. Who is correct?

a. A, because B having foreclosed on the first mortgage, no further action is allowed including the
foreclosure of the 2nd mortgage. (Cruz v Filipinas, Recto Law)
b. B, because he had the right to foreclose on both mortgages since they were purposely made in
connection of the sale. However, he can't avail of the other remedies anymore.
c. A, because B having foreclosed on the first mortgage, he already bound himself to that action.
d. B, because he can exhaust all the remedies until full payment of price can be acquired.

OBLICON MCQ

1. When the debtor binds himself to pay when his means permit him to do so, the obligation is:

a. Conditional
b. Pure
c. Simple
d. With a Period

ANSWER: D

2. Contracts which cannot be sued upon unless ratified, thus it is as if they have no effect yet are:

a. Voidable
b. Rescissible
c. Void
d. Unenforceable

ANSWER: D

3. If the obligation of the debtor is "I will pay you my debt after I have arrived from abroad," this is
a. Unenforceable
b. With a Period
c. Void
d. Conditional

ANSWER: D

4. When the characters of the creditor and the debtor are merged in one and the same person,
there is extinguishment of the obligation by:

a. Compensation
b. Merger of Rights
c. Novation
d. Remission

ANSWER: B

5. Through insidious words or machinations, A was able to induce B to enter into a contract which
without them B would not have agreed to it. There is:

a. Undue Influence
b. Fraud
c. Mistake
d. Misrepresentation

ANSWER: B

6. "A sells to B his lot and house in the city if A decides to transfer and live in the countryside" is
an example of:

a. Mixed Condition
b. Potestative Condition
c. Casual Condition
d. Resolutory Condition

ANSWER: B

7. It is a mode of extinguishing an obligation when two persons in their own right are creditors of
each other.

a. Confusion
b. Reformation
c. Compensation
d. Novation
ANSWER: C

8. A contract is in the stage of conception when:

a. There is meeting of the minds.


b. Negotiations are in progress.
c. The parties come to an agreement.
d. The contract is perfected.

ANSWER: B

9. If the obligor binds himself to perform his obligation as soon as "he shall have obtained a loan"
from a certain bank, this obligation is:

a. With a Term
b. Conditional
c. Suspensive
d. Resolutory

ANSWER: B

10. Contracts entered into in a state of drunkenness or during a hypnotic spell are:

a. Void
b. Valid
c. Voidable
d. Legal

ANSWER: C

11. Delay in the giving or delivering of a thing

a. Mora​ ​solvendi ex re
b. Mora solvendi ex persona
c. Mora accipiende ex re
d. Mora accipiende ex persona

ANSWER: A

12. Which of the following statements is false?

a. Obligations to give definite things and those that are not susceptible of partial performance shall be
deemed divisible.
b. Execution of a certain number of days of work shall be divisible.
c. Accomplishment of work by metrical units are divisible
d. An obligation to pay a certain amount in ten annual installments is divisible.

ANSWER: A
13. ​This contract is without effect unless ratified:

a. Marriage between first degree cousins


b. Contract of sale between two insane persons
c. Contract of sale between husband and wife
d. Donation between husband and wife

ANSWER: B

14. Which of the following contracts is not void ab initio?

a. Those whose object is outside the commerce of men


b. That whose object did not exist at the time of transaction
c. That which contemplates an impossible service
d. That which is undertaken in fraud of creditors

ANSWER: D

15. ​Rescission of contract can take place in this case

a. When the thing which is the object of the contract is legally in the possession of a third person who
acted in bad faith
b. When he who demands rescission can return whatever he may be obliged to restore
c. When the party seeking resolution can perform only as to part and as to remainder
d. When the seller cannot return the installments paid to him by the buyer

ANSWER: B

16. A defective contract where damage or lesion is essential

a. Rescissible
b. Voidable
c. Unenforceable
d. Void
ANSWER: A

17. ​An instrument may be reformed d

a. Simple donations inter vivos wherein no condition is imposed


b. Wills
c. When the instrument does not express the true intention of the parties due to mistake
d. When the real agreement is void

ANSWER: C

18. ​Three of the following contracts are void. Which one is not?

a. Oral contract of partnership of three partners and capital contribution is more than P3,000 in cash
b. Written contract contemplating impossible services
c. Oral contract of partnership where real estate is contributed as capital
d. Agent's authority to sell land is given orally.

ANSWER: A

19. ​When a third person assumes the payment of the obligation even without the knowledge and
consent of the debtor but with the consent of the creditor

a. There is novation
b. There is delegation if debtor is released
c. There is subrogation
d. There is expromission if debtor is released

ANSWER: D

20. ​Which of the following is not an element of legal compensation?

a. Debts to be compensated are due and demandable


b. There is controversy or adverse claim over any debts to be compensated
c. There are two or more debts of the same kind
d. There are two or more persons who are creditor or debtors of each other.

ANSWER: B

Question: Why is it important that obligation in contractsbe faithfully fulfiiled??

Answer: ​Laws are enacted to have order (or require the performance of a particular action to achieve the
same) Since the terms and conditions of a contract are considered the governing laws between parties, it
is necessary to faithfully fulfill one's obligation, that is, to avoid the occurrence of dispute. Hence, why do
we have to fulfill our obligations, that is to achieving order between the parties to the contract.

Question: How do you apply the laws on obligations and contracts to everyday activities and
business dealings?​
Answer​: We apply laws on obligations and contracts unconsciously to our daily activities. One will not
notice that in performing a simple common task you have applied several principles on obligations and
contracts. Like, for example, buying a piece of pandesal, the buyer will offer to buy and the seller will
accept. At this point, once the seller and buyer agrees to the object and price, and their minds have met;
each of them will not just rescind or refuse to comply. Even without knowledge of law, one will not just
back out from the perfected sale.

Then, after the perfection of the contract of sell; unknowingly, they will go on to consummate the same.
The buyer will hand the money and the seller, in return, will hand over the pandesal. A clear example of
reciprocal obligations, isn't?

If the money given is more than the price of the pandesal, the seller will give the change. That is because
he knows no one should be enriched at the expense of another; hence, he has the obligation to return
what is not due him. Likewise, if what was delivered to the buyer is more than what he paid for, he will
return the same under the same principle, creating an implied obligation to return.

In a simple, but very common, transaction, the parties are not aware that they are applying the basic
principles of law on obligations and contracts. They might not be well versed, or even, have not had any
formal education, yet they apply these simple principles of law unconsciously. Thus, If you ask: how we
apply laws on obligations and contracts on our daily activities, we apply it unknowingly, but instinctively,
depending on one's value.

SAMPLE PROBLEMS WITH SUGGESTED ANSWERS.

1. D is obliged to deliver 5 bags of powder soap to C 7 days from their agreement. On due date, D
delivered 5 bags of powder soap mixed with chalk. What is the status of the agreement between D and
C?
* The agreement is valid. The fraud was committed during the performance of the obligation and
​ ot causal fraud
not during the agreement of the parties. This is a case of incidental fraud (​dolo incidente) n
(dolo causante).

2. D is obliged to give C a specific watch, a specific ring or a specific bracelet. The parties agreed that C
will have the right to choose the thing which will be given to him. Before C could make his choice, the
watch and the ring are lost through D’s fault, successively. What is the right of C?
* C may choose the delivery to him of the bracelet, or the price of the watch or the price of the
ring plus damages.

3. A, B, C and D are obliged to give X, Y and Z P12,000. X may collect from A how much?
* P1,000. When the obligation is silent, it is presumed joint.

4. A, B, C and D, joint debtors are obliged to give X, Y, and Z, solidary creditors, P12,000. How much may
X collect from A?
* P 3,000. As a solidary creditor, X may collect the whole amount owed by the joint debtor A.

5. A, B, C and D, solidary debtors, are obliged to give X, Y and Z, joint creditors, P12,000. How much may
X collect from A?
* P4,000. As a joint debtor X is entitled only to his proportionate share, and A being a solidary
debtor may be required to pay the said amount.
6. A, B, C and D, solidary debtors, are obliged to give X, Y and Z, solidary creditors, P12,000. How much
may X collect from A?
* P12,000. X being a solidarity creditor may ask for the payment of the whole amount in behalf of
his co-creditors subject to a responsibility of X to give the latter their corresponding shares. Similarly, A as
a solidary debtor may be required to pay the whole amount of the obligation subject to reimbursement
from his co-debtors.

7. A and B are indebted to X and Y for P10,000. A and B share in the debt in the ratio of 1:3 while X and
Y share in the credit in the ratio of 2:3.

a. How much may X collect from A if the debtors are joint debtors, while the creditors are joint
creditors?
* P1,000. The obligation is joint on both the debtor and creditor, therefore there are as
many debts (credits) as debtors (creditors).

COMPUTATION:
​Proportionate Share of A:
¼ x P10,000 = P 2,500

Proportionate Share of B on A’s Proportionate Debt:


2/5 X P2,500= P 1,000
b. How much may X collect from A if there is active solidarity?
* P2,500(see computation above). The whole amount of the proportionate share of A
could be collected by X. As a solidary creditor, X may collect the whole amount of the obligation
corresponding against one or more debtors.

c. How much may X collect from A if there is passive solidarity?


* P4,000. Being a joint creditor, X can collect only his corresponding share in the credit.
The full amount could be collected to A, being a solidary debtor.

COMPUTATION:
​Proportionate Share of X in the credit:
2/5 x P10,000 = P 4,000

d. How much may X collect from A if there is mixed solidarity?


P10,000. Since there is mixed solidarity the whole amount of obligation may be collected
by any of the solidary creditors against any of the solidary debtors.

8. A, B and C are solidary debtors of X, Y and Z, solidary creditors, in the amount of P2,700. X renounces
the whole obligation without the consent of Y and Z. The debtors accepted the renunciation. What is the
legal effect of the renunciation?
* The whole obligation is extinguished, however X shall be liable to the corresponding shares of
the other co-creditors as they have agreed upon.

9. A, B and C are solidary debtors of X in the amount of P3,000. X renounces the share of A and A
accepts the renunciation. Thereafter B becomes insolvent. What is the legal effect of the renunciation?
* A will be liable for P500, while C will be liable for P1,500 (P1,000 + P500). Since the remaining
obligation is P2,000 after the renunciation of A’s share, and thereafter B becomes insolvent, A and C
would have to absorb the debt corresponding to B in the amount of P1,000. This shall be divided equally
by A and C.

10. A, B and C are solidary debtors of X in the amount of P30,000. C was insane at the time the obligation
was constituted. What is the legal effect?
* X may collect from either A or C P20,000. Art. 1222 provides that a solidary debtor may avail
himself of the partial defense of the insanity of C. Such defense is personal to C and would therefore
affect only the part of the debt to which C may be responsible.

11. A, B and C are obliged to deliver a specific horse to X, Y and Z. What would be the legal effect when
C cannot comply with his obligation?
*This is a case of a jointly-indivisible obligation. Assuming there was a valid demand made
against all the debtors and since C could not comply with his part of the obligation, the obligation is
converted into a monetary obligation to pay the value of the horse plus damages. (1224)

12. D borrowed P10,000 from C with G as guarantor. Subsequently, D paid C P 2,000. Unknown to D, T a
third person paid C P10,000 believing that D still owed C such amount. What is the legal effect of the
payment by T?
* T can recover P8,000 from D, the law provides that only the amount to which the creditor has
benefited could be reimbursed to the third person paying if the payment was without knowledge of the
creditor. If D cannot pay, T cannot go after G to collect. (1236)

13. D owes C P10,000. T offers to pay D’s obligation and tells D that D need not reimburse him. However,
D does not give his consent to T’s offer not to be reimbursed. C, nonetheless, accepts the payment from
T. Was the payment valid?
* The payment is valid insofar as C is concerned. The case is deemed to be a donation, however
to be constituted as such D’s consent is necessary. (1238)

14. D obtained a loan of P10,000 from C who was in his right mind at the time he granted the loan. On
due date, D paid his obligation of P10,000 to C who had since become insane. C lost P4,000 of the
amount he received and spent P6,000 for his food and other necessary expenses. Was the obligation
extinguished?
* The obligation is extinguished up to P6,000 only. Payment of an obligation to an incapacitated
person shall be valid when the person has kept the thing delivered and only insofar as the payment has
been beneficial to him. (1241)

15. D owes C the following debts: P4,000 due on May 1; P4,000 due on May 8; P4,000 due on May 15;
P4,000 due on May 22; P4,000 due on May 29; and P4,000 due on June 5. The debts represented the
price of magazines which were delivered to D on a weekly basis. Of the six debts, the one due on May 22
is secured by a pledge of D’s ring. By agreement of the parties, C may demand payment even before the
due date of the debt. As of May 31, D had not paid any of the six debts. On May 31, D wanted to make
payment but he had only P4,000. How is the payment to be applied?
* Since no agreement as to which debts should be settled first and absent the fact that the
creditor has not issued any receipt applying the amount to any of the debts to which the debtor has not
protested and there was no cause of invalidating the same, and the period is for the benefit of the creditor
then the P4,000 should be applied to the one due on May 22 since it is most burdensome.

16. Refer to no. 15 Assuming that D did not designate the debt to be paid when he remitted the amount of
P4,000 to C on May 31. C issued a receipt for the payment he received from D but he did not also
designate the debt that was being paid. How would the payment be applied?
* The payment shall be applied to the debt due on May 22 since it is the most onerous.

17. D owes C the following debts: P200,000 due on June 1, 2008 secured by a car; P200,000 due on May
1, 2008 with 10% interest; P300,000 due on May 1,2008 with 8% interest; P300,000 due on May 1,2008
secured by house and lot; P200,000 due on April 1,2008 and P300,000 due on March 1, 2008. By
agreement of the parties, C may demand payment even before the due date of the debt. Today is May 1,
2008 D had not paid any of the six debts. D wanted to make payment but he had only P1,000,000. How is
the payment to be applied?
* Since no agreement as to which debts should be settled first and absent the fact that the
creditor has not issued any receipt applying the amount to any of the debts to which the debtor has not
protested and there was no cause of invalidating the same, furthermore the period is for the benefit of the
creditor, the P1,000,000 should be applied as follows:

Amount available P 1,000,000

Debt secured by house and lot ​300,000

Balance 800,000

Debt with 10% Interest

Interest 10% x P 200,000= P20,000


Principal ​200,000 ​ 220,000

Balance 580,000

Debt with 8% interest

Interest 8% x P 300,000= 24,000


Principal ​300,000 ​324,000

Balance 256,000

****
The remaining P 256,000 shall be distributed proportionately to the debts that are due and demandable
as follows:

Remaining Debts: Ratio % Share (P256,000​)


(a)Mar 1 P 300,000 3/5 (300/500) 60% P 153,600
(b)Apr 1 ​P 200,000 ​ 2/5 (200/500) 40% 102,400
500,000

18. Francis, husband and Mitch, wife are legally separated. By order of the court which decreed the legal
separation, Francis is obliged to give a monthly support of P20,000 to Mitch payable in advance within the
first five days of the month. Mitch owes Francis P20,000 by way of loan. On the other hand, Francis has
not yet given Mitch’s support for P20,000 for the preceding month and another P20,000 for the present
month. All the debts are due. Can compensation take place?
* Yes. If Mitch demands her support for the current month, Francis may claim compensation as
regards the loan that Mitch owes him.

19. D owes C P20,000 with G as Guarantor. C, on the other hand, owes D, P15,000. Both debts are
already due but D is insolvent. Can compensation take place?
* Yes. A guarantor can set up compensation as regards what the creditor may owe the principal
debtor. However, C may still collect from G P5,000.

20. D owes C P20,000 due on June 20. C owes D P14,000 due on June 15. On June 12, C assigned his
right to X. D consented to such assignment without any reservation of his right to the compensation. On
June 20, how much may X collect from D?
* P20,000. No compensation shall take place since there was no reservation of such right by D
upon the assignment of the credit. (1285 par 1)

21. D owes C P30,000 due on June 20. C owes D the following debts: P15,000 due on June 2; P4,000
due on June 14; P6,000 due on June 16; and P3,000 due on June 18. On June 17, C assigned his right to
X. C notified D of the assignment but D did not give his consent thereto. How much may X collect from D
on June 20?
* P5,000. Since the assignment, although with knowledge, is without consent of D therefore
compensation shall take place but only to the debts prior to the assignment. (1285 par 2)

22. Refer to no. 21. Assume the same facts except that when C made the assignment of his credit to X, C
did not notify D about it. It was only on June 20 when X went to D to collect that D learned of the
assignment. How much may X collect from D?
* P2,000. The assignment is without knowledge of D, therefore the latter may set up
compensation of all credits prior to the same and also later ones until he had knowledge of the
assignment. (1285 par 3)

23. D owes C P50,000. Subsequently, D proposed to C that T will assume his (D’s) debt. C accepted the
proposal of D. On due date, T could not pay because of his insolvency which was in fact existing but was
not known to D or of public knowledge at the time that he delegated his debt. Can C hold D liable?
*C cannot hold D liable because his (D’s) obligation was extinguished when he was substituted
by T. (1295)

24. D obliged himself to give 10 grams of shabu to C. Later, the parties agreed that D would instead give
to C 10 sacks of rice. Is the novation valid?
* Novation is void because the original obligation is void, therefore there is nothing to novate.
Hence, C cannot demand the delivery of 10 sacks of rice from D.

25. S sold his cabinet to B for P5,000. The parties gave their consent freely to the contract before its
perfection. After delivery of the cabinet to B and his payment of the price to S, B was informed by N, his
neighbour that B paid too much for the cabinet since he (N), knew of a similar item that is sold for a lower
amount. B now wants to set aside the contract because he believed he got a bad bargain. Decide.
* B may not set aside the contract based on the principle of Mutuality of Contract.

26. A and B entered into a joint venture contract whereby B agreed to put up a restaurant on the lot of A.
N, A’s neighbour who owned a lot across A’s lot, learned of the transaction between A and B. Anticipating
that many customers would patronize the restaurant, N decided to improve his vacant lot for parking. He
incurred P20,000 for a guardhouse and other improvements which he had not yet paid to his contractor.
Later, however, A and B mutually cancelled their earlier contract and entered into a new one whereby B
agreed to put up the restaurant on another lot belonging to A which was located about 100 meters from
the original area. N learned of the cancellation of the contract and decided that he would sue A and B for
damages he allegedly sustained by reason threof. Decide.
* A is not entitled to sue A and B for the damages he sustained. This is not a case of a stipulation
pour autrui, where the parties in a contract deliberately conferred upon a third person a favor or benefit.
The benefit that would have been received by N from the contract between A and B was only incidental
and did not give him the right to recover damages.

27. D obtained a loan from C amounting to P200,000. To secure the loan, D mortgaged his lot. C
registered the mortgage with the Register of Deeds. Later, D sold the lot to X who was not aware that the
lot was mortgaged. On the due date of the loan, C demanded payment but D could not pay. Accordingly,
C decided to foreclose the mortgage on the lot. X, however opposed the foreclosure claiming that he was
not bound by the mortgage since he was not a party thereto. Decide.
* X was bound by the mortgage. Subsequent purchaser is bound by the mortgage registered in
the Register of Deeds even if he is not aware of it. The contract herein is one which creates real rights
therefore third persons who come into possession of the object of contract is bound thereby. (1312)
28. S sold his only horse to B for P30,000. The parties agreed that S shall deliver the horse one week
from the execution of their agreement. B, however, should pay the price immediately and in certified
check. In the place of S and B, it was the custom that anyone selling a horse should place a horseshoe
on its feet. Is the seller bound to place a horseshoe on the horse?
* S is obliged to place a horseshoe on the horse because the observance of custom or usage is a
consequence of entering into a contract. (1315)

29. On June 1, S offered to sell a specific generator set to B for P300,000. B sent his letter of acceptance
to S on June 8. On June 10, however, S became insolvent. On June 12, S received the letter of
acceptance. Was the contract perfected?
* The contract was not perfected because the insolvency of S occurred before he came to learn
of the acceptance of his offer. It is well settled rule that an offer becomes ineffective upon the insolvency
of either party before acceptance is conveyed. (1323)

30. P gave a special power of attorney to A to sell P’s house and lot for P2M. On May 7, A, pursuant to
the authority granted to him by P, offered to sell the house and lot to B at the price of P2M. B accepted
the offer on May 8 by sending a letter of acceptance to A on such date, which letter of acceptance was
received by A on May 10. On May 11, P died before A could inform him of B’s acceptance. Was the
contract perfected?
* The contract was perfected on May 10 when A received the letter of acceptance. An offer made
through an agent is accepted from the time acceptance is communicated to him. (1322)

31. On May 1, S offered to sell his car for P500,000 to B who was interested in buying the same. In his
letter to B, S stated that he was giving B up to May 31 to decide whether to buy the car or not. On May
25, S personally went to B to inform him that he was no longer willing to sell the car unless the price was
increased to P600,000 because another buyer was interested in buying the car for the said amount of
P600,000. May S validly withdraw his offer?
*S may validly withdraw his offer to B and all that S needs to do is to inform B of such withdrawal.
Such withdrawal must be made before the lapse of the period of acceptance, unless there is an option
founded upon a consideration, as something paid or promised. (1324)

32. B purchased 100 pieces of notebook from S at P41.95. When B reached home, he discovered that
the invoice showed a total amount due of P4,915 instead of P4,195. Is the contract valid?
*The contract is valid. The mistake is a simple mistake of account and does not render the
consent of the parties vitiated. This shall only give rise to correction. Mistake to vitiate consent must refer
to the substance of the thing which is the object of the contract or to a principal condition which moved the
parties to enter into a contract (1331) *The excess P720.00 should be returned by B.*

33. D has been for more than 10 years the personal physician of P, a sickly man of 70 years. D talked to
P almost everyday in the course of giving medical service to P to convince P to donate his lot to D since D
told him that he was the only one who could take care of him. Because of the persistence of D, P finally
signed the deed of donation of the lot in D’s favor with all the formalities required by law. Is the deed of
donation valid?
*The deed of donation is voidable on the ground of undue influence. (1337)

34. A, who knew that his ring was embellished with glass, told B that the embellishment was emerald. B,
who knew that his watch was gold-plated, told A that it was made of pure gold. Believing that A’s ring was
embellished with emerald and A, believing that B’s watch was made of pure gold, then entered into a
contract whereby they exchanged their respective articles. A week later, B discovered that the ring was
adorned only with an ordinary glass. Can B ask for annulment of the contract?
* Neither party can ask for annulment since both are guilty of fraud. The contract therefore is
valid. In order that a contract may be considered voidable and be annulled fraud must not be committed
by both parties. (1344)

35. S and B entered into a contract where they made it appear that S was selling his lot and building to B.
The truth, however was that S was donating his lot and building to B. What is the status of the contract?
* The contract is a relative simulated contract. Therefore the parties are bound by the contract of
donation, their true intention, since no third person is prejudiced and their purpose is not contrary to law,
moral, good customs, public order or public policy. (1346)

36. S and B orally entered into a contract whereby S sold his one-year production of mangoes to B for
P100,000. B gave a downpayment of P20,000 for which S issued a receipt. What is the status of the
contract between S and B?
* The contract is valid. Future things may be the object of contracts. (1347 par 1)

37. A died leaving properties estimated at P1M to his sons S and T. Subsequently, S sold through a
private instrument one-half of his inheritance to B for P300,000, although his share was still to be
delivered. What is the status of the contract?
*The contract is valid since the inheritance is an existing inheritance. (1347 par 2)

38. S sold his car to B for P100,000 so that he can have money to purchase shabu. B knew nothing of the
motive of S. Was the contract valid?
*The contract is valid because the illegal motive of S does not affect its validity. Motive is different
from the cause of the contract. (1351)

39. D and C entered into a contract wherein D agreed to give to C P50,000 within 30 days from the date
of the execution of their agreement, which however does not state the consideration received by D from
C. What is the status of the contract?
*The contract is valid because the cause is presumed to exist and is lawful unless the contrary is
proved by the debtor. (1354)

40. G, the guardian of M, a minor sold the fish harvested from the fishpond of M for P20,000. The fish,
however had a value of P30,000. What is the status of the sale?
*The sale is rescissible because M suffered the minimum amount of lesion required by law to
make the contract rescissible. (1/4 x 30,000= 7,500, the lesion is 10,000). (1381 par 1)

41. D owes the following creditors: X, P50,000; Y, P60,000; and Z, P90,000. He has assets valued at
P400,000. Subsequently, D donated, among his assets, a parcel of land valued at P250,000 to C. The
donation and acceptance were made in a public instrument. What is the status of the donation of the land
made by D to C?
* The donation is rescissible. It is presumed to have been made in fraud of creditors. The
remaining value of the assets after donation would be insufficient to cover the debts contracted by D
before the donation. (1387 par 1)

42. C filed a complaint in court against D to collect a money debt amounting to P500,000. After due
hearing, the court rendered judgment in favor of C. Shortly after the rendition of the judgment and before
C has collected D’s debt, D sold a parcel of land to X. Z, another creditor, learned of the sale made by D
to X and now files an action to rescind the sale. Who has a better right to rescind the sale of D to X?
*Both C and Z have a right to rescind the sale since the sale is presumed in fraud of creditors
having been made by the party against whom judgment has been rendered.(1387 par 2

43. R, the representative of A, an absentee sold the corn with a value of P30,000 and the palay with a
value of P50,000, harvested from A’s agricultural farm for a total price of P50,000. A, whose domicile was
subsequently known, was informed of the sale made by G. May A seek rescission?
*A may seek rescission of the total sale to recover the damages he suffered. (1381 par 2)

44. S, 17 ½ years old sold his bicycle to B, 24, for P12,000. The price is payable in 12 monthly
instalments. After reaching 18, S continued to collect the remaining six installments until the price was
paid in full. Can S annul the contract?
*S may no longer annul the contract. There was implied ratification when S upon reaching the
age of majority continued to collect the remaining instalments. (1393)

45. M, the manager of an electric company offered to by a one-square meter lot from S for P400. M
showed S his power of attorney executed by the electric company and duly signed by the board of
directors. M informed S that the lot would be the site of an electric post which the electric company would
erect. S agreed to the odder of M. M agreed to come back the following day to give the company check
for the purchase price as well as the written contract. When M returned to the place of S, S refused to
sign the contract and accept the check. S told M that at any rate, M or the electric company could not
enforce the contract since it was not in writing. Is S correct?
*The contract must be in writing for it to be enforceable against S. This is a sale of real property.

46. On June 1, 2005, B, a businessman, met by chance C, a building contractor at the lobby of a hotel.
Over a cup of coffee, B informed C that he would be needing the services of C for the construction of a
three-storey building that B was planning to put up beginning August 1, 2006. B added that he would be
paying C the amount of P4M for the construction. C agreed to B’s proposal. In the meantime, B gave C a
check for P500,000 representing the downpayment on the contract price. Except for the table napkin on
which B made some sketches of the building, no written contract was signed by the parties. On August 1,
2006, B called up C to inform him that the construction would begin within the next two weeks. C told B
that he would no longer want to proceed with the contract and that he would be returning the
downpayment. May B enforce the contract?
*B may enforce the contract against C although there was no formal agreement signed by the
parties because C was deemed to have ratified the contract by his acceptance of the downpayment.

47. D was checking out of a hotel when he found out that he had lost his wallet. Since he had no money
to pay his bills, he texted G, a businessman friend to help him. Upon reading the text message, G texted
M, the hotel manager, who knew G since G was a frequent guest at the hotel whenever he visited the
area. The text message read: “Please let D leave. If he does not pay, I will be the one to pay.” No other
information was contained in the text message. Thereafter, D signed a promissory note in favor of the
hotel. Is the promise of G enforceable?
*If D cannot pay, M cannot enforce the promise of G since G’s promise is not the writing required
by law. (1403 2b)

48. S, insane, orally sold his radio to B, 17 years old for P400. When T, the mother of S learned about the
sale, she asked S to give to her the amount that he had received. S obediently gave the amount of P400
which his mother used to buy their food. What is the status of the contract?
*The contract is voidable because the mother of S is deemed to have ratified the contract. (1407)

49. Francis promised to give P10,000 by way of support to Mitch, if Mitch agrees to live with Francis as
his wife without the benefit of marriage. Mitch because she truly loves Francis accepted his promise. After
6 months, because Mitch was seen with Edgar, her ex-boyfriend, Francis left their home and never came
back. Can Mitch now still receive the support promised by Francis?
*The contract between Francis and Mitch is illegal and constitutes an offense, therefore it is void.
50. B, a law student informed O that he needed a bicycle which he would use in going to and from San
Beda College of Law-Alabang. O told B that he may get the bicycle in O’s room. Was there a perfected
contract?
* Yes. The contract is one of commodatum, where one party loaned something to another. In this
case, the bicycle was loaned by O to B.
Obligations & Contracts Name: ______________________________________
Mid-term Examinations February 1, 2014
Instructions​: Write your name on the blank provided above. Your answers should be written in your test
booklet in ​chronological order​. If you want to erase your answer, simply place a straight one bar line
over the answer. You may use this questionnaire as your scratch paper. Write legibly.

Multiple Choice Questions: Write the letter corresponding to the correct answer. Do Not Forget to place
the number before the correct answer.

1. Which among the following may not be an underlying philosophy in obligations arising from
Contracts?
a) Reciprocity
b) Freedom to contract
c) Good faith
d) Law between the parties

2. If the obligor is unable to comply with his obligation by reason of fortuitous event the obligation is
extinguished, except when
a) He is guilty of negligence
b) When the obligations to give is indeterminate
c) When the obligation is determinate
d) When the obligation proceeds from a criminal offense

3. Lessor A leased his apartment unit to Lessee B. In the contract of lease, it was stipulated therein
that for failure of he lessee B to pay his rentals for two (2) consecutive months, A may validly
eject B, without resort to court. Upon B’s failure to pay 2 monthly rentals he was ejected from the
premises, B now goes to court for Damages for his alleged failure to exercise his day in court
since he was ejected from the premises without a court hearing. Decide..
a) B’s contention is valid because the contract between vests reciprocal obligations, hence he
must have his right to seek judicial recourse
b) B’s contention is valid because it must be proved that he indeed failed to pay 2 installments;
c) A’s action is right because the injured party has the right to rescind the obligation
d) A’s action is correct because A’s non resort to judicial action is expressed in the contract;
e) B’s contention is correct because in all reciprocal obligations, the right of the parties to
rescind the contract is implied and that judicial decree is needed.

4. In an obligation whereby the obligor is bound by different prestations,


a) It is the debtor who has the right of choice unless the creditor is given that right
b) The right of choice to the debtor gives him the freedom to chose
c) The debtor can chose to give a portion or part of each prestations except those which are
indivisible

5. Mode of Extinguishing obligations whereby the right of subrogation may occur:


a) Novation f) Resolutory
b) Compensation g) Loss of the Thing due
c) Condonation h) Compromise
d) Payment i) Death of one of the parties in personal obligations
e) Confusion j) Application of Payment
6. Mode of Extinguishing obligation which is subject to a condition the fulfilment of which
extinguishes the obligation:
a) Novation f)​ Expiration or fulfillment of resolutory term
b) Compensation g) Loss of the Thing due
c) Condonation h) Compromise
d) Payment i) Death of one of the parties in personal obligations
e) Confusion j) Application of Payment

7. Mode of Exntinguishing obligation whereby there are several debts to one creditor and the debts
are of the same kind and burden, ​the payment (not sufficient) is applied to all debts
proportionately. (​Sobrang give away ng clue​)
a) Novation f) Expiration of resolutory term
b) Compensation g) Loss of the Thing due
c) Condonation h) Compromise
d) Payment i) Death of one of the parties in personal obligations
e) Confusion j) ​Application of Payment

Page 02.

8. When a third person substitutes to the debtor without the latter’s knowledge and the third person
is unable to fulfill the obligation, extinguishes the obligation of the old debtor.

a) Novation f) Expiration of resolutory term


b) Compensation g) Loss of the Thing due
c) Condonation h) Compromise
d) Payment i) Death of one of the parties in personal obligations
e) Confusion j) Application of Payment

9. The distinction between negligence and fraud is


a) Voluntariness
b) Intention
c) Degree of Damages sustained by the injured party
d) Negligence or culpa is a civil in character while dolo or fraud is criminal in essence.

10. The principle giving remedies to the creditor for his protection and enforcement of his right
against the debtor such as exhaustion of debtor’s property in his possession, to be subrogated to
all the rights and actions of the debtor and third to impugn all the acts of the debtor, which were
intended to defraud him is ​not​ applicable in
a) Obligations with suspensive condition
b) Obligations with resolutory condition
c) The case of fruits and interests
d) Rights which are purely personal

Potrebbero piacerti anche